Income Tax Review Questions Flashcards Preview

Income Tax > Income Tax Review Questions > Flashcards

Flashcards in Income Tax Review Questions Deck (198)
Loading flashcards...
1
Q

Nicholas had the following stock transactions this year:
- Nicholas purchased stock last year on October 31 for $12,000. On March 5 of this year the stock became completely worthless.
- Nicholas purchased Section 1244 stock several years ago for $200,000. He sold the stock in the current year for $100,000.
What is Nicholas’ net long-term loss for the current year?
a) $12,000
b) $50,000
c) $62,000
d) $112,000

A

c) $62,000
- The worthless stock is long-term because the stock is considered to be worthless as of the last day of the year in which it became worthless. Section 1244 allows the taxpayer to deduct up to $50,000 of loss on 1244 stock against ordinary income. The remaining amount will be capital loss; in this case, the loss is long-term. Therefore, there is $62,000 of LT loss ($12,000 + $50,000)

2
Q

Morgan had a machine she used in her business. She is downsizing the business and needs a smaller machine. A local business owner, Caleb, has offered to enter into an exchange of machines. Morgan will receive Caleb’s machine worth $75,000 and $25,000 in cash in exchange for her machine worth $100,000. Morgan’s basis in her machine is $80,000 and Caleb’s basis is $25,000. What is Morgan’s basis in the new machine?

a) $25,000
b) $75,000
c) $80,000
d) $100,000

A

b) $75,000
- In a like-kind exchange, the property exchanged must be real estate. This is a traditional transaction so tax-payer will pay gain on realization. FMV of asset and cash is $100k, basis $80k, taxable gain $20k, basis of new asset is FMV of asset $75k

3
Q

Destiny’s business property was destroyed by fire. The original basis was $250,000. She received insurance proceeds of $600,000. She acquired replacement property two months later and paid $620,000 for the new property. What is Destiny’s basis in the new property?

a) $250,000
b) $270,000
c) $600,000
d) $620,000

A

b) $270,000
- In an involuntary conversion, the basis of the new property is generally a carryover basis. The basis is adjusted up if additional dollars are invested, and adjusted down if the replacement asset fair market value is less then the asset’s adjusted basis. $250,000 + $20,000 additional investment.

4
Q

Sydney and Kevin were married last year. They decided to live in Kevin’s home after they were married. Kevin has lived and owned the home for 5 years. Sydney has lived there for 9 months. They have sold the home because Kevin’s job transferred him to another state. Kevin’s basis in the house is $400,000 and the fair market value is $1,000,000. What is the gain that will be recognized on the sale of the home?

a) $100,000
b) $256,250
c) $412,500
d) $600,000

A

b) $256,250
- Each spouse is entitled to a $250,000 exclusion of gain if they meet the ownership and use rules. Only one spouse is required to own the home for 2 of the last 5 years, however each is required to use the home for 2 out of the last five years. If the time frame is not met, a partial exclusion is available if the move was due to job change or other unforeseen circumstances. In this case, Kevin qualifies for the full $250,000 exclusion. Sydney is entitled to 9/24 of the exclusion = $93,750. The gain is $1,000,000 – $400,000 basis – $250,000 Kevin’s exclusion - $93,750 Sydney’s exclusion = $256,250

5
Q

Which of the following is a Section 1231 asset?

a) Machine used in a business, purchased 6 months ago.
b) Land used in a business, purchased 10 years ago.
c) Inventory of a business.
d) Accounts receivables of a business

A

b) Land used in a business, purchased 10 years ago.
- A Section 1231 asset is depreciable or real property used in a trade or business held greater than one year. Inventory and accounts receivables are ordinary income assets. The machine was purchased less than 1 year ago.

6
Q

Jose’s company placed a machine into service several years ago. The company has taken depreciation deductions of $40,000. The original purchase price was $90,000. If the company sells the machine today for $120,000 how much ordinary income will be recognized?

a) $0
b) $30,000
c) $40,000
d) $70,000

A

c) $40,000
- The property is Section 1231 property because it is depreciable property used in a trade or business held greater than one year. It is subject to Section 1245 recapture. Ordinary income will be recognized to the extent depreciation was taken ($40,000) and the remaining gain is Section 1231 gain ($30,000) taxed at 15%

7
Q

Victoria’s company purchased business real estate many years ago that is used in her business. The purchase price of the property was $1,000,000. The company has taken depreciation deductions of $800,000, $500,000 of which is attributable to straight line depreciation. If the company sells the property today for $2,000,000 what is the tax consequence?

a) $1,800,000 taxed at 20%
b) $300,000 taxed at ordinary income rates, $500,000 taxed at 25% and $1,000,000 taxed at 20%
c) $1,500,000 taxed at 20%
d) $500,000 taxed at 25% and $1,000,000 taxed at 20%

A

b) $300,000 taxed at ordinary income rates, $500,000 taxed at 25% and $1,000,000 taxed at 20%
- The property is Section 1231 property because it is real property used in a trade or business held greater than one year. The property is subject to Section 1250 recapture. The amount of accelerated depreciation ($300,000) will be recaptured as ordinary income, the straight line portion ($500,000) will be taxed at 25%, and the remaining gain ($1,000,000) will be treated as Section 1231 gain taxed at 20%.

8
Q

Ben sold some land to his brother Logan on January 1 of the current year. Ben had a basis in the land of $40,000. Ben sold the land to Logan for $240,000. Logan could not afford to pay for the land outright so Ben agreed to sell the land using an installment sale. Logan will pay $2,000 per month for 10 years plus an applicable rate of interest. Six months after Logan purchased the land, Logan sold the property for $300,000. How much gain will Ben recognize in the current year if 12 payments were made?

a) $4,000
b) $20,000
c) $200,000
d) $260,000

A

c) $200,000
- Since Ben and Logan are related parties, the entire gain must be recognized in the current year because Logan sold the property within 2 years. Therefore, Ben must recognize gain of $200,000. Otherwise, the installment treatment would have been allowed. The installment treatment would have been: (Total gain/ sales price) x payments = recognized gain. ($200,000 / $240,000) x $24,000 = $20,000.

9
Q

Matthew had the following income and expenses for the current year:
- Salary - $30,000
- State of Nevada bond interest - $4,000
- Gift of property from his uncle – Fair market value: $30,000; Uncle’s basis: $3,000
- Student loan interest - $5,000
- Medical expenses - $6,000
- Roth IRA contribution - $2,000
Calculate Matthew’s AGI for the current year.
a) $19,000
b) $25,000
c) $27,500
d) $31,500

A

c) $27,500
- Salary of $30,000 - $2,500 student loan interest = $27,500
- The State of Nevada Bond’s are municipal bonds and are not taxable. The gift is not taxable. The medical expenses are itemized deductions. The Roth IRA contribution is not deductible. Student loan interest is only deductible up to $2,500.

10
Q

Madison had the following income and expenses in the current year.
- Salary - $22,000
- Child support received - $6,000
- Alimony received from 2016 divorce-$10,000
- Personal injury award from an auto accident. She lost the use of her left hand and was awarded damages of $200,000. She also received $50,000 in punitive damages and $30,000 for mental distress, of which $5,000 was used for counseling.
- Paid property tax of - $3,000
- Paid personal residence interest of - $9,000
Calculate Madison’s gross income for the current year.
a) $82,000
b) $95,000
c) $107,000
d) $285,000

A

c) $107,000
- Salary of $22,000 + Alimony of $10,000 + $50,000 in punitive damages and $25,000 in mental distress = $107,000
- The child support is not taxable. The personal injury award is not taxable; however, the punitive damages are. The award for mental distress is only excluded to the extent the money was actually used to pay medical bills. The property tax and residence interest are itemized deductions. Alimony is taxable since agreement is pre 12/31/18

11
Q

Mark had a very bad year. He recently learned that his wife, Abigail embezzled $300,000 from her employer this year. The embezzlement has not yet been identified by the employer, and Abigail has no intention of paying it back. After this occurred, Mark went into deep depression and began using illegal drugs. For a short time, Mark sold illegal drugs to maintain his habit. He estimates that he earned roughly, $40,000 selling drugs. However, he spent $30,000 manufacturing them and $2,000 in bribes to police officers. After a run-in with a local gang, Mark decided he better straighten up. He and Abigail began couples’ counseling to work on their problems. Although it cost them $7,000 in medical expenses, Mark was able to determine why Abigail had resorted to embezzlement. Apparently, she had been gambling on the weekends and had lost $250,000. She claims the thrill of winning kept her in the game, although she only won $2,500. Assuming Mark and Abigail file married filing jointly, what is their AGI?

a) $42,500
b) $53,500
c) $312,500
d) $340,000

A

c) $312,500
- $300,000 in embezzled funds + $40,000 in illegal drug sales - $30,000 in COGS + $2,500 in gambling winnings = $312,500
- For illegal drug business, he must include the income but he may deduct the cost of goods sold. The bribes are against public policy and may not be deducted.

12
Q

Ethan, age 17, had total income of $20,000 for 2019 - $10,000 was from his paper route, $3,000 was from local city bonds, and $7,000 was dividends from investments. How much of his income will be taxed to him at his rate?

a) $1,850
b) $4,800
c) $10,000
d) $13,650

A

a) $1,850
- $10,000 + 7,000 = $17,000 in gross income. The local bonds are municipal bonds and are not taxable. His standard deduction is the greater of $1,100 or earned income plus $350 limited to $12,200; in this case $10,350 is his standard deduction.
Step 1 - work with the unearned income: $7,000 - the $1,100 standard deduction for unearned income, leaving $5,900. The next $1,100 of unearned income is taxed at the child’s rate (single tax filer rate), leaving $4,800 to be taxed at the trust rate.
Step 2 - work with the earned income: $10,000 - the remaining standard deduction of $9,250 (10,350 - 1,100 used against unearned income), leaving $1,850 taxed at the child’s rate. In total, Ethan will have $10,350 (1,100 + 9,250) of standard deduction, $1,850 (1,100 + 750) taxed at the child’s rate and $4,800 taxed at the trust rate.

13
Q

Which of the following individuals must file a tax return in 2019?

a) Isabella and Andrew file married filing joint. Isabella is age 32 and Andrew is 62. They have income of $16,000.
b) Ava is single and has income of $17,000.
c) Samantha is age 70. She supports her 8-year-old granddaughter. The granddaughter lives with Samantha. Samantha earned $14,000.
d) Hannah is age 65 and has Social Security benefits of $18,000. She has no other income

A

b) Ava is single and has income of $17,000.
- Isabella and Andrew need income of $24,400 or less to avoid filing. Ava needs income of $12,200 or less to avoid filing. Samantha needs income of $20,000 or less to avoid filing. Notice she can use the Head of Household standard deduction of $18,350 since she is caring for a qualifying child, (however, that is not taken into account when determining whether she has to file). She also gets an additional standard deduction of $1,650 for being 65 or older. Hannah does not have any taxable income because her Social Security benefits are below the limit.

14
Q
Ashley had the following income and expenses in the current year: 
- Salary - $60,000 
- Child support received - $10,000 
- Alimony received - $20,000. This is her third year of alimony (divorce 2016). The first year she received $100,000 and the second year she received $50,000. 
- Traditional IRA contribution - $4,000 
- Casualty loss - $8,000 
What is Ashley’s AGI? 
a) $0 
b) $3,500 
c) $68,000 
d) $80,000
A

b) $3,500
- The alimony will be included in the gross income. In this case she will have alimony recapture in year 3 because of the greater-than-$15,000 decrease between years 1 and 2 and years 2 and 3. The recapture short-cut formula is R3 = P1 + P2 - 2(P3) – $37,500. Therefore, R3 = $100,000 + $50,000 – $40,000 – $37,500 = $72,500.
Salary of $60,000 + alimony received of $20,000 – alimony recapture of $72,500 – IRA contribution of $4,000 = $3,500.
The child support is not included and the casualty loss is an itemized deduction. Since the contract predates 12/31/18, it is subject to the recapture rules.

15
Q

David has a vacation home that he rents for 200 days per year. He uses it for his own enjoyment for 15 days per year and his children use the property for 15 days. The property is considered:

a) Primary personal
b) Mixed use property
c) Primary rental
d) 1245 property

A

b) Mixed use property
- The use of David and his children are added together to determine David’s use. He rents the property greater than 14 days so the property can’t be personal. David’s use is more than 10% of the rental days, thus, it can’t be rental property. Therefore, it must be mixed use. Section 1245 property is Section 1231 personalty that is subject to depreciation

16
Q

Sarah had the following losses in the current year (no disaster declaration by the President):
- Termite damage to her home. The fair market value of the home before the damage was $250,000. The fair market value after the damage was $225,000. Sarah’s basis in the property is $200,000.
- Her car was broken into and they stole her camera. The camera had a basis of $1,500 and a fair market value of $1,000. She received $200 in insurance proceeds.
- There was a fire on Sarah’s boat. The fire caused a $10,000 decline in value. Sarah’s basis in the property was 6,000. Sarah received $1,000 in insurance proceeds.
Sarah’s AGI is $30,000. What is her available casualty loss for the current year?
a) $0
b) $2,700
c) $27,600
d) $31,300

A

a) $0
- The termite damage is by statute not a casualty loss. Without a disaster declaration casualty losses are no longer deductible (TCJA 2017).

17
Q

Alyssa, a self-employed CFP® professional, wanted to go to Hawaii for vacation. Being very creative, she decided to schedule several business meetings while she was there in order to deduct the expense as a business trip. She left her home on Friday, scheduled three business meetings for the following Wednesday and returned home the following Monday. Which of the following is true?

a) She cannot deduct anything as a business expense.
b) She can deduct one night of hotel and one day of meals as a business expense.
c) She can deduct all of the travel, one night of hotel and one day of meals as a business expense.
d) She can deduct all of the travel, hotel and meals as a business expense.

A

b) She can deduct one night of hotel and one day of meals as a business expense.
- This is a domestic trip. Therefore, the travel is only deductible if the trip is primarily for business. Clearly this trip is for pleasure; therefore, the travel is not deductible. The one night of hotel and the one day of meals will be deductible since she did work for one day.

18
Q

Natalie and her husband James have been married for 25 years. They have three children, Chloe, age 18, John, age 16, and Brianna, age 8. Natalie and James have a combined salary of $150,000. During the year the family had the following expenses:
- Brianna attends a private boarding school for gifted children. She has an extremely high IQ. The cost of the school is $40,000 and includes lodging and meals.
- John had an illegal drug problem during the year. He attended a drug rehabilitation program which cost $5,000.
- Chloe gave birth to a baby, named Nathan after Chloe’s grandfather. The cost of the birth was $9,000. The father of the baby has not been involved; Natalie and James provide all of the support for the new baby.
- After Nathan was born, the family spent $600 in bottled water to mix with formula, $2,000 on a diaper service and $200 on nonprescription drugs
What is Natalie and James’ taxable income for 2019?
a) $111,700
b) $111,950
c) $113,000
d) $125,600

A

d) $125,600
- This question is complicated because you must determine whether they can itemize or use the standard deduction. The school was not medically necessary. If they select the standard deduction, their taxable income would be $150,000 – $24,400 = $125,600.

19
Q

Jason was injured in an accident. He is now a quadriplegic. He installed an elevator in his home so he could reach the second floor. The cost of the elevator was $10,000 and the increase in value attributable to the elevator was $4,000. He also had exit ramps built, the hallways widened and bathroom fixtures lowered at a cost of $20,000. The increase in value to the home was $1,000. Jason’s income for the year is $25,000. What is Jason’s available deduction for medical expenses?

a) $0
b) $22,475
c) $23,500
d) $27,500

A

c) $23,500
- Capital improvements can be deducted as a medical expense. The general rule is that the expense less the increase in value to the home is an eligible deduction. However, certain expenses that allow a physically handicapped person live independently are not reduced by the increase in the home’s value. This includes exit ramps, widening of hallways, and lowering of bathroom fixtures but not elevators. Thus the medical deduction is: ($10,000 - $4,000 +20,000) – (10% x 25,000) = $23,500

20
Q

Which of the following taxes paid can be deducted as an itemized deduction?

a) FICA tax withheld by the employer
b) Gift tax paid
c) State excise tax
d) State use tax

A

d) State use tax
- State use tax can be deducted as an itemized deduction. The rest cannot be deducted. An example of a state excise tax is a gasoline tax.

21
Q

Samuel had the following inflows and outflows in the current year.

  • Salary of $40,000
  • Income distributed to him from a trust fund of $250,000
  • Investment income of $4,000.
  • Qualified student loan interest of $5,000.
  • Investment expenses of $400.
  • Investment interest expense of $5,000
  • Mortgage interest of $200,000. The loan on the personal residence is $2,000,000. Which of the following statements is true?
    a) The student loan interest is not deductible.
    b) The mortgage interest is deductible in full.
    c) The investment interest expense is deductible in full.
    d) The income from the trust will not be taxable to Samuel.
A

a) The student loan interest is not deductible.
- Distributions of income from trusts are taxed to the beneficiary. Therefore, he will be over the phased out limitation for the student loan deduction. Investment interest expense is limited to net investment income, which is $3,600 ($4,000 – $400). Personal residence interest deduction is limited to the interest on the first $750,000 in home acquisition indebtedness.

22
Q

Ella received a wonderful painting when her grandmother died several years ago. The grandmother’s basis was $2,000 and the date of death value was $75,000. Ella decided to give her painting to the local museum of art so that everyone could enjoy its beauty. The painting was worth $100,000 at the time of the donation. Ella’s AGI is $100,000. What is the most Ella could take as a charitable deduction in the current year?

a) $25,000
b) $30,000
c) $37,500
d) $50,000

A

d) $50,000
- Ella has two choices. She can elect basis and deduct up to 50% of her AGI, or she can elect to use the fair market value and deduct up to 30% of her AGI. Her basis in the painting is $75,000, the grandmother’s date of death value. If she elects basis then she can deduct $50,000, and if she elects FMV then she can deduct $30,000. The question asks for the greatest deduction for the current year. Note that if it said the greatest deduction total, then the FMV election would be more appropriate because she would be able to deduct more over the long run.

23
Q

Taylor has wages of $60,000. She has a 20% investment in rental real estate in which she actively participates. She has basis in the investment of $100,000. The investment allocated a loss for the current year of $75,000. What is the suspended loss due to at-risk rules for the current year?

a) $0
b) $25,000
c) $50,000
d) $75,000

A

a) $0
- Since she has $100,000 in basis and only a $75,000 loss there is not a suspended loss due to at-risk rules. There is, however, a suspended loss due to passive activity rules. The suspended loss is $50,000 because she is allowed to take $25,000 of loss against ordinary income in the current year because she is an active participant in a rental real estate activity and her income is below the $100,000 threshold

24
Q

ABC, Inc. grants Noah one ISO on January 1, YR1. The exercise price is $15. The market price on the exercise date (January 1, YR2) is $35. What is the AMT consequence when Noah exercises the ISO?

a) $0 AMT gain
b) $15 AMT gain
c) $20 AMT gain
d) $35 AMT gain

A

c) $20 AMT gain
- The AMT gain is the difference between the market price and the exercise price (known as the bargain element) at the date of exercise. $35 – $15 = $20

25
Q

Which of the following is true regarding AMT?

a) If Kayla had deductible medical expenses for regular tax purposes of $10,000, she will have no adjustment for AMT purposes.
b) Interest for home acquisition indebtedness deducted for regular tax purposes must be added back.
c) Charitable contributions deducted for regular income tax purposes are limited to 30%.
d) Municipal bond interest must be added back for AMT purposes.

A

a) If Kayla had deductible medical expenses for regular tax purposes of $10,000, she will have no adjustment for AMT purposes.
- Interest deducted for home acquisition indebtedness is not an add-back for AMT; however, it is if the loan is a home equity loan. Charitable contributions are not added back for AMT purposes. Municipal bonds in general are not added back for AMT purposes; however, private activity bonds are

26
Q

Which of the following is true?

a) Any unused adoption credit available may be carried forward for up to 5 years.
b) The child tax credit is available for any child under the age of 19.
c) The child and dependent care credit is not available for expenses paid to relatives caring for a child.
d) The definition of an eligible child is the same for the adoption expense credit, the child tax credit, and the child and dependent care credit.

A

a) Any unused adoption credit available may be carried forward for up to 5 years.
- The child tax credit is available for any child under the age of 17. The child and dependent care credit is available for expenses paid to relatives caring for a child as long as the relative is not a child under the age of 19 of the taxpayer. The definition of an eligible child is different for the adoption expense credit, the child tax credit, and the child and dependent care credit

27
Q

Dawn and Jamie live in a community property state. They own a personal residence with a basis of $500,000 and a fair market value of $800,000, and personal property with a basis of $300,000 and a fair market value of $100,000. They also own two cars, one with a basis of $50,000 and a fair market value of $20,000 and the other with a basis of $40,000 and a fair market value of $30,000. Assume Jamie died in 2015 and left all of his assets to Dawn. If Dawn sells the personal residence in 2019 for $1,000,000, what is her realized gain?

a) $200,000
b) $350,000
c) $500,000
d) $750,000

A

a) $200,000
- When Jamie dies, Dawn receives a step-to-fair-market value in both halves of the personal residence. There-fore, her new basis is $800,000. Her gain at the sale is $200,000 ($1,000,000 – $800,000).

28
Q

Savannah received a gift of property from her mother, Cindy. The property was worth $100,000 at the date of transfer. Cindy did not pay any gift tax and the annual exclusion did not apply to this gift. Cindy had inherited the property from her father Ralph. Ralph had a basis in the property of $5,000 and the fair market value at the time of Ralph’s death was $50,000. What is Savannah’s basis in the property?

a) $5,000
b) $50,000
c) $100,000
d) It is unknown until Savannah sells the property.

A

b) $50,000

- Savannah’s basis = Cindy’s basis. Cindy’s basis is the fair market value at her father’s death.

29
Q

Anna has salary of $50,000 and two investments, Limited Partnership Investment A and B. She does not materially participate in either investment. Her basis in the partnerships are: LLP A - $50,000; LLP B - $25,000. LLP A had a $75,000 gain in the current year and LLP B had a $100,000 loss. What is the net gain or loss for the two investments that will be recognized on the current tax return?

a) $3,000 loss
b) $25,000 loss
c) $0 gain or loss
d) $50,000 gain

A

d) $50,000 gain
- The investments are passive activities. Therefore, the loss is limited to the at-risk amount and then passive income. The loss for LLP B is limited to $25,000 since that is the at-risk amount. It can then be netted against LLP A for a total of $50,000 gain ($75,000 – $25,000).

30
Q

Emily recently filed her tax return. She failed to file a proper extension and filed 70 days after the due date. Along with her return she remitted a check for $5,000. What is Emily’s failure to file penalty?

a) $75
b) $450
c) $675
d) $750

A

c) $675
- The failure to file penalty is: (5% (further reduced by .5% failure to pay penalty) x 3 months x $5,000) – $75 = $675 - The failure to pay penalty is: 0.5% x 3 months x $5,000 = $75 Total penalty = $750

31
Q

Emma’s husband, Joshua, died last year. Before his death, Joshua exercised a substantial amount of ISOs. Before they could pay the tax, the stock tanked leaving them no cash to pay the AMT that resulted from the exercise. Emma would like to challenge the amount due in court, as she has already exhausted the IRS pro-cess. Her lawyer believes that the US Court of Appeals has unfavorable precedent for her transaction. The lawyer also believes that the facts of the case may help Emma if she has a sympathetic audience. However, she is unable to pay the claim currently and her top priority is to utilize a court that does not require her to pay up-front. She does want to be able to appeal if the lower court does not render in her favor. Which court is most appropriate for Emma?

a) US Tax Court
b) US District Court
c) US Court of Federal Claims
d) US Tax Court: Small Claims Division

A

a) US Tax Court
- Since Emma’s top priority is that she can’t pay, the Tax Court and Small Claims Division are the only options. We do not know if she would qualify for the Small Claims Division (claims less than $50,000), but since she wants to be able to appeal, this is not an option either. There is not a jury trial available here like there is in district court, but again, she states not paying is the top priority. The US Court of Appeals is the court that the US Court of Federal Claims appeals to.

32
Q

Which of the following cannot be relied upon for legal precedence?

  1. Proposed Regulations
  2. Revenue Rulings
  3. Private Letter Rulings by someone other than the requester.
  4. Temporary Regulations

a) 1 and 3
b) 2 and 4
c) 2 and 3
d) 1, 2, and 3

A

a) 1 and 3
- Proposed Regulations and Private Letter Rulings cannot be relied upon for legal precedence. Both Revenue Rulings and Temporary Regulations may be relied upon.

33
Q

BestBusiness, LLC bought two machines in the last two years. They bought the first machine on December 28 of last year. The machine arrived in the plant on January 2 of the current year and it was installed on February 15 of the current year. The purchase price was $65,000. The second machine was purchased on November 15 and delivered and installed on December 3. The purchase price was $35,000. Which depreciation convention should they use?

a) Half-year
b) Mid-quarter
c) Mid-month
d) They can choose between half-year and mid-quarter.

A

a) Half-year
- They should use half-year. This is the default for personal property placed into service. The mid-month convention is used for real property. The mid-quarter convention would be used if greater than 40% of personal property is placed into service in the last quarter. In this example, the first machine is not placed into service until the current year because it was not actually installed until February 15. Therefore, 65% was placed into service in the first quarter and only 35% was placed in the last quarter.

34
Q

Holly purchased a building for $100,000 and paid a commission of $5,000. She made building improvements of $20,000 and took a cost recovery amount of $12,000 in the first year. At the end of the first year, what is Holly’s adjusted basis in the building?

a) $125,000
b) $113,000
c) $105,000
d) $88,000

A

b) $113,000
- Holly’s original basis in the building was the purchase price ($100,000) plus the commission ($5,000) plus the improvements ($20,000) for a total basis of $125,000. The cost recovery amount, however, reduces Holly’s basis by $12,000 for an adjusted basis at the end of the year of $113,000.

35
Q

Jill purchased office equipment for $20,000, paid sales tax of $1,400, and shipping costs of $3,000. This is the only Section 179 property that Jill has put into service this year, and she has taxable income of $100,000. What is the maximum Section 179 deduction that Jill can take this year?

a) $20,000
b) $21,400
c) $23,000
d) $24,400

A

d) $24,400

- Jill may elect to expense the entire cost of placing her new office equipment into service ($24,400).

36
Q

Jake is a general partner in Smoothie Queen, a general partnership. Jake received a K-1 from the partnership, which contained the following items:
Partnership Taxable Income = $200,000
Dividend Income = $2,500
Long-Term Capital Gain (on investments) = $6,000
How much self-employment tax will Jake have to pay in the current year?
a) Jake is not required to pay self-employment tax.
b) $14,130
c) $15,772
d) $21,835.90

A

d) $21,835.90
- Although Jake must report all items of partnership income, he is only required to pay self-employment tax on the partnership taxable income of $200,000. He would incur $21,129 in self-employment tax, calculated as follows:

Net Earnings from SE = $200,000
x 92.35%
= Amount subject to SE tax = $184,700

SS Portion (12.4% x $132,900) = $16,479.60
+ Medicare Portion (2.9% x $184,700) = $5,356.30
Total SE Taxes = $21,835.90

37
Q

Your client is contemplating the exchange of two parcels of investment land for two similar parcels. Given the following details of the proposed transactions, compute the amount of recognized gain and loss (if any) on both parcels if your client does the exchanges.
• Parcel A: Ten acres of land acquired 15 years ago with a current basis of $50,000. In exchange your client will receive eight acres of land (FMV $80,000) and $20,000 of cash.
• Parcel B: Twenty acres of land acquired two years ago with a current basis of $100,000. In exchange your client will receive twelve acres of land (FMV $75,000) and $10,000 of cash.
Parcel A Recognized Gain; Parcel B Recognized Loss
a) $20,000; $0
b) $20,000; $10,000
c) $50,000; $10,000
d) $20,000; $15,000
e) $50,000; $15,000

A

a) $20,000; $0
- A1 must be worth $100,000 because A2 + $20,000 = $100,000. The client must recognize gain up to the boot received $20,000
- The client will receive economic value = $85,000 ($75,000 + $10,000). The client will not recognize loss due to Section 1031

38
Q

A client was divorced in 2017. The client’s ex-spouse has custody of their 10-year-old daughter. During the current year, with agreement from the ex-spouse, your client made alimony payments of $12,000 directly to contractors and maintenance workers for upkeep of the ex-spouse’s house. In addition, the client paid child support payments of $6,000 to the ex-spouse. Your client’s adjusted gross income before any deductions for the above-listed expenses was $75,000. What is the appropriate amount of the deduction on your client’s federal income tax return in the current year?

a) $0.
b) $6,000.
c) $12,000.
d) $18,000.

A

c) $12,000.
- Divorces finalized prior to 12/31/2018 are grandfathered in the old rules, deductible to the payor and taxable to the payee. With agreement of the ex-spouses, alimony can be redirected to home maintenance and maintain it alimony status.
- Payments for child support are not considered to be alimony and are not deductible.

39
Q

Three investors wish to start a manufacturing business. The business is expected to generate a large income which it will reinvest for many years. Investor #1 has substantial assets which he plans to contribute to the business. Investor #1 is also concerned about showing too much business income on his personal return. Which business structure(s) would be most appropriate for the business?

  1. A limited partnership with Investor #1 as the limited partner.
  2. A business trust with all three as equal interests.
  3. An S corporation with all three as equal shareholders.
  4. A C corporation with all three as shareholders.

a) 1, 2 and 3.
b) 1 and 3.
c) 2 and 4.
d) 4 only.
e) 1, 2, 3 and 4

A

d) 4 only.

- Only the C corporation will solve investor #1’s concern about income on his personal return

40
Q

Which is the best source for obtaining information about the intent of a very recent change in the tax law?

a) RIA Federal Tax Coordinator.
b) Congressional Committee Reports.
c) Treasury Regulations.
d) Tax Court Reports.
e) US Master Tax Guide.

A

b) Congressional Committee Reports.
- The question asks about very recent changes to tax law, and the best source is Congressional Committee reports for bills passed and signed into law. Recent changes would not be reflected in the other sources of information.

41
Q

Your client, a wealthy physician in the top marginal tax bracket, is interested in purchasing a franchise in a fast-growing chain with some of his colleagues. After carefully reviewing the proposal, you have deter-mined that apart from a large up-front investment, the business will not need to retain income and the income generated in subsequent years will be paid out to the investors. Furthermore, your client wants to be assured that after investing so large an amount, the business would not be disrupted if one of his partners lost interest or encountered personal financial reversals. What form of business makes the most sense given these circumstances?

a) Limited Partnership.
b) General Partnership.
c) C corporation.
d) Professional Corporation.
e) S corporation.

A

e) S corporation.
- The question asks about legal forms. Partnerships (A and B) are excluded due to disruption if a partner were to leave. Corporation (C) is excluded due to implicit double taxation and no need for capital accumulation. Professional Corporation (D) is not available for a franchise operation. Therefore, the S corporation is the best choice - a flow-through entity, offering limited liability and ease of continuation and exchange of ownership. A Limited Liability Corporation may well have accomplished the same goals had it been a choice.

42
Q

Jeffrey and Karen Jones have given cash gifts to their children over the years.

  • Mark, age 13, earns $2,500 in salary.
  • Jennifer, age 18, earns $5,000 in dividends and capital gains.
  • Nancy, age 12, earns $2,500 in dividends and interest.
  • Steven, age 10, earns $900 in dividends and interest. Whose income is subject to the tax at the trust tax rate?
    a) Steven’s.
    b) Jennifer and Nancy’s.
    c) Nancy’s.
    d) Steven, Jennifer and Nancy’s.
    e) Nancy and Mark’s.
A

b) Jennifer and Nancy’s.
- The question relates to the kiddie tax, which applies to unearned income in excess of $2,200 by a child under the age of 19. E is excluded, because Mark, while under the age of 19, has only earned income. D is excluded, because Steven, age 10, had unearned income less than $2,200. Therefore, the answer is B, because Nancy, age 12, had unearned income over $2,200 and Jennifer is under 19 with unearned income over $2,200.

43
Q

Which of the following incomes is/are not taxed under Social Security self-employment tax?

  1. Rental real estate income.
  2. Small part-time repair shop income.
  3. Shareholder’s share of S corporation’s income in excess of salary.
  4. Income of an individual working as an independent contractor.
    a) 1, 2 and 3 only.
    b) 1 and 3 only.
    c) 2 and 4 only.
    d) 4 only.
    e) 1, 2, 3 and 4
A

b) 1 and 3 only.
- Income from rental property is not subject to Social Security tax nor are dividends to S corporation share-holders/employees as long as employees’ compensation is at or above FMV. Statements #2 and #4 are subject to FICA.

44
Q

A minority nonemployee shareholder in an S corporation

  1. receives compensation when the corporation declares a dividend.
  2. votes for the Board of Directors at the annual shareholders’ meeting.
  3. receives a K-1 annually in order to prepare a personal income tax return.
  4. reports on a personal income tax return a pro rata share of corporate profit or loss.
    a) 1, 2 and 3 only.
    b) 1 and 3 only.
    c) 2 and 4 only.
    d) 4 only.
    e) 1, 2, 3 and 4.
A

e) 1, 2, 3 and 4.
- Statements #1, #2, #3, and #4 describe an S corporation nonemployee shareholder; therefore, E is the correct answer. Statement #1 uses the term “compensation” to describe the flow of funds, not to describe the concept of taxable income.

45
Q

A client purchased a mutual fund with a $10,000 lump-sum amount four years ago. During the four years, $4,000 of dividends were reinvested. Today the shares are valued at $20,000 (including any shares purchased with dividends). If the client sells shares equal to $13,000, which statement(s) is/are correct?

  1. The taxable gain can be based on an average cost per share.
  2. The client can choose which shares to sell, thereby controlling the taxable gain.
  3. To minimize the taxable gain today, the client would sell shares with the higher cost basis.
  4. The client will not have a gain as long as he/she sells less than what he/she invested.
    a) 1, 2 and 3 only.
    b) 1 and 3 only.
    c) 2 and 4 only.
    d) 4 only.
    e) 1, 2, 3 and 4
A

a) 1, 2 and 3 only.

- The choice is the client’s if he maintains adequate identification records. Statement #4 is clearly incorrect.

46
Q

A client sold an apartment building last year for $100,000, paying a sales commission of $5,000 plus $2,500 closing costs. Twenty years ago, the building’s original cost was $80,000. Total straight line depreciation of $40,000 had been taken. The building had a mortgage of $60,000 that was assumed by the buyer. What is the seller’s adjusted cost basis?

a) $32,500.
b) $37,500.
c) $40,000.
d) $52,500.

A

c) $40,000.
- The seller’s cost basis is equal to the purchase price minus the depreciation ($80,000 - $40,000 = $40,000). The commission and closing costs reduce the net sales price.

47
Q

Which of the following statements about S corporations is true?

a) S corporation status is automatic if there are fewer than 100 shareholders.
b) S corporations are prohibited from having more than one class of stock.
c) S corporations are prohibited from earning passive income.
d) S corporations may have nonresident aliens as shareholders.

A

b) S corporations are prohibited from having more than one class of stock.
- A is incorrect because S corporation status is never automatic. S corporations must file Form 2553 by the 15th of the third month after the beginning of the corporation for S status to be in effect for the first year. S corporations are not permitted to have more than 100 shareholders. More than one class of stock is not permitted with an S corporation, but the one class of shares can have voting shares and non-voting shares. Finally, nonresident aliens are not permitted to be shareholders of S corporation stock.

48
Q
Your client is contemplating the sale of some of her holdings in her employer’s stock. The stock was acquired in four separate purchases as follows:
Date; Shares; Cost per Share; Cost
6/1/86; 200; $10; $2,000
6/1/88; 200; $18; $3,600
6/1/90; 200; $12; $2,400
6/1/92; 200; $20; $4,000
Total; 800; $12,000
She wants to know the least amount of gain she would be required to report if she sold 500 shares for $12,500. Compute this gain. 
a) $500. 
b) $3,700. 
c) $4,300. 
d) $5,000. 
e) $5,700
A

b) $3,700.
- The client can use specific identification as a method to determine the basis of stock for purposes of calculating gains and losses. This method will generally produce the least amount of gain, because the highest basis shares can be sold first.
Sale price (500 shares) $12,500
Sell 200 shares @ $20/share -$4,000
Sell 200 shares @ $18/share -$3,600
Sell 100 shares @ $12/share - $1,200

12,500
- $8,800
$3,700 gain

49
Q

A client, who is a sole proprietor, is going to give her best customers gifts as a token of her appreciation. If the client wants to deduct the full cost of the gifts, what is the maximum amount that she may spend on each customer’s gift?

a) $25.
b) $50.
c) $100.
d) $250.

A

a) $25.

- $25 is the limit for business gifts per donee per year.

50
Q

Which of the following dispositions of IRC Section 1245 recapture property would result in the immediate recapture of some or all of previous depreciation deductions?

a) A distribution by a partnership to its partners.
b) A donation to a charity.
c) A disposition at death.
d) A sale for cash and an interest-bearing note.

A

d) A sale for cash and an interest-bearing note.
- A represents a transaction that entails a carryover basis. B represents a transaction that entails a carryover basis, unless boot is received. C represents the adjustment in basis to FMV at death. D represents a sale for cash that causes recapture of gain to the extent of gain in excess of basis. Any gain over the original purchase price would be Section 1231 gain.
- Note: A sale at adjusted taxable basis would not cause recapture. However, D is the best answer presuming the sale is for an amount in excess of adjusted taxable basis.

51
Q

John is a contractor who has just purchased a tractor for use in his business. John paid $25,000 plus $1,250 in sales tax for the tractor. The local municipality also imposes an annual personal property tax of $500. The tractor has an expected useful life of 5 years. What is John’s basis in the tractor for depreciation purposes?

a) $25,000.
b) $25,500.
c) $26,250.
d) $26,750.

A

c) $26,250.
- Basis of an asset is equal to the purchase price plus any addition to place the property into service. $25,000 + $1,250 = $26,250 The personal property tax is an annual executory cost and is expensed annually, rather than being capitalized.

52
Q

Your client, who has a taxable income of $200,000, is concerned about being subject to the alternative mini-mum tax (AMT). The following income and deductions were included in computing taxable income. Select the one item that may be added to (or subtracted from) regular taxable income in calculating the AMT.

a) A long-term capital gain of $100,000.
b) A cash contribution to your client’s church of $25,000.
c) Dividend income of $100,000.
d) Qualified residential interest of $25,000.
e) A state income tax payment of $25,000.

A

e) A state income tax payment of $25,000.
- B, C, and D are incorrect. E is correct since no taxes are deductible for AMT purposes. Do not let A confuse you; it is included both in regular tax and AMT.

53
Q

Alisha Sheridan, a CFP® professional and fee-only financial planner, has assisted Roger Regate, a self-employed physician, in tax and investment planning during the year. Identify the schedule(s) on which Alisha’s fee may be deductible by Roger on his federal income tax return.

  1. Schedule A–itemized deductions.
  2. Schedule C–profit or loss from business.
  3. Schedule D–capital gains and losses.
    a) 1 only.
    b) 2 only.
    c) 3 only.
    d) 1 and 2 only.
    e) 1, 2, and 3
A

b) 2 only.
- Tax planning for the business is deductible on Schedule C, as is business investment planning. However, usually the CFP® professional is performing services for the individual, which is no longer deductible on Schedule A. Since the question is not definitive as to the services and the word “may” (very permissive) is used, D is the best answer. At no time would these expenses be included on Schedule D, which is used to reflect capital gains and losses.

54
Q

Five years ago, Tom Mohy bought 10,000 shares of stock at $10 per share in a pharmaceutical company. Today, the stock is worth $200,000 and is paying a dividend of $8,000 per year. Tom feels that the stock will continue to appreciate at a rate of 12% per year, including the dividend. Tom wants to establish a college education fund for his two daughters, ages 19 and 9. Which of the following statements is/are true?

  1. If Tom gives 2,500 shares of stock to his 19-year-old daughter, all dividends from the 2,500 shares will be taxed in her income tax bracket.
  2. If Tom gives 2,500 shares of stock to his 9-year-old daughter, all dividends from the 2,500 shares will be taxed at her marginal rate.
  3. Two years from now, if Tom’s older daughter sells her 2,500 shares of stock at $30 per share, Tom will need to report the gain as a long-term capital gain on his personal income tax return.
  4. All dividend income earned by his 9-year-old daughter which exceeds $2,200 in 2019 will be taxed at trust tax rate.
    a) 2 only.
    b) 1 and 2 only.
    c) 1 and 3 only.
    d) 1 and 4 only.
    e) 3 and 4 only.
A

d) 1 and 4 only.
- Statements #1 and #4 are correct.
- Statement #1 is true because it doesn’t tell us that she meets the exception of under “24” and a student. If she is in school, then the kiddie tax rules would apply until age 23. Statement #4 would be true for 2019 since his 9-year-old daughter would be subject to the kiddie tax with unearned income over $2,200.

55
Q

Which of the following imposed the first constitutional federal income tax?

a) Revenue Act of 1861.
b) 16th Amendment.
c) Revenue Act of 1916.
d) None of the above.

A

d) None of the above.
- Answer “A” is incorrect because although the Revenue Act of 1861 did impose a federal income tax, it was later found to be unconstitutional because Congress did not have the power to levy an individual income tax at that time. Answer “B” is incorrect because the 16th Amendment gave Congress the power to impose an individual income tax, but did not itself impose that tax. Answer “C” is incorrect because the Revenue Act of 1916 raised the rates previously imposed under the Revenue Act of 1913. Therefore, answer “D” is correct because the Revenue Act of 1913 imposed the first constitutional income tax.

56
Q

Which is the best source for obtaining a plain language understanding about the current tax law?

a) Commerce Clearing House Federal Tax Guide.
b) Congressional Tax Committee Reports.
c) Treasury Regulations.
d) Tax Court Reports.

A

a) Commerce Clearing House Federal Tax Guide.
- Option “A” is correct because Commerce Clearing House (CCH) provides plain language interpretation of tax law. Option “B” is incorrect as the Congressional Committee Reports (sometimes known as the Blue Book) provides congressional reasoning for enacting tax law. This language is often very technical and difficult to understand. Option “D” is incorrect because Tax Court Reports provide rulings of the U.S. Tax Court in the form of case law.

57
Q

Which of the following statements regarding deductible expenses is not true?

a) Above-the-line deductions are sometimes referred to as adjustments.
b) Above-the-line deductions are usually considered to be more favorable to the taxpayer than below-the-line deductions.
c) Below-the-line deductions are usually considered to be more favorable to the taxpayer than above-the-line deductions.
d) Taxpayers may take the greater of their itemized deductions or the standard deduction.

A

c) Below-the-line deductions are usually considered to be more favorable to the taxpayer than above-the-line deductions.
- Above the line deductions (for AGI) are more favorable since many phaseouts are based on AGI. A lower AGI may result in additional deductions or tax credits. The question was looking for the false statement.

58
Q

Jason has three capital transactions for the current year:

Short-term capital loss of $5,000

Short-term capital gain of $3,000

Long-term capital loss of $2,000

What is the net effect on Jason’s taxes if he is in the 35% tax bracket?

a) $1,400 tax reduction
b) $1,050 tax reduction
c) $850 tax reduction
d) $450 tax reduction

A

b) $1,050 tax reduction
- Net the STCG and STCL = $2,000 STCL.
- The $2,000 LTCL plus the $2,000 STCL = Total Loss of $4,000.
- He can only utilize $3,000 to offset ordinary income at 35% = $3,000 × 0.35 = $1,050. The remaining $1,000 is a long-term capital loss carryover.

59
Q

As a direct result of the rules under TCJA 2017, qualifying dividends will be treated in which manner:

a) Qualifying dividends are taxed in the same way as capital gains at an 18% rate for those in the 28% and higher marginal tax bracket.
b) Qualifying dividends are taxed at a newly instituted 5% tax rate.
c) The taxation on dividends have not been impacted under TCJA.
d) Qualifying dividends are taxed at set dollar breakpoints.

A

d) Qualifying dividends are taxed at set dollar breakpoints.
- Under prior law, the capital gain breakpoints were related to the tax breakpoints. Under TCJA the capital gain breakpoints are at set dollar amounts not corresponding to the current tax brackets. See provided tax tables in Helpful Documents within Blackboard.

60
Q

Noah has been working part-time through college and earned $20,000 last year with a total federal income tax liability of $1,200. This year he will earn $100,000 with an expected income tax liability of $15,000. What is the lowest amount of tax withholding Noah should have to meet the safe harbor rules?

$1,200
$12,000
$13,500
$15,000

A

$1,200
- He has two choices: 100% of last year’s tax liability or 90% of this year’s tax liability. Last year’s income tax liability was $1,200 and 90% of this year is $13,500. Therefore, the lowest amount that of tax withholding that needs to be met is $1,200.

61
Q

Ginger, age 21 and a full-time student for a degree at State University, qualifies as a dependent on her parents’ return. During the summer, she earned $5,500 from a part-time job. Her only other income consisted of $950 interest on a savings account. What is Ginger’s taxable income for the current year?

$0
$600
$1,050
$5,500

A

$600
- The standard deduction for Ginger is the greater of $1,100 or $350 plus earned income but not to exceed the normal standard deduction. Therefore $350 + $5,500 = $5,850 so it is not limited in 2019. The total income is $5,500 + $950 = $6,450. Taxable income is $6,450 - $5,850 = $600.

62
Q

Abner and Bette have been married for 20 years and have always filed a joint return. They never itemize their deductions. Abner and Bette have gross income of $80,000 and deductions for adjusted gross income in the amount of $5,000, and they do not have any children. Neither Abner nor Bette are over the age of 65 and neither is blind. What is Abner and Bette’s taxable income in the current year?

$50,600
$56,000
$57,000
$75,000

A

$50,600
- Abner and Bette’s taxable income is equal to their gross income less deductions for adjusted gross income less the greater of the standard deduction or itemized deductions, and less personal and dependency exemptions. Therefore, their taxable income can be calculated as follows:

Gross Income $80,000

Less Deductions for AGI ($5,000)

Adjusted Gross Income $75,000

Less Standard Deduction ($24,400)

Taxable Income $50,600

63
Q

Julian purchased 100 shares of Home Depot, a domestic corporation, common stock on July 7th this year. The ex-dividend date for their quarterly dividend is July 12th. Julian sells the Home Depot stock on August 15th of this year. If Home Depot paid a dividend of $10 on Julian’s 100 shares, what are the tax consequences to Julian if Julian is in the 25% tax bracket?

No tax liability since the qualified dividend rate is 0% for individuals in the 25% bracket.
$1.50 of tax liability since the qualified dividend rate is 15% for individuals in the 25% bracket.
$2.50 of tax liability since the ordinary dividend rate is the taxpayer’s marginal rate.
Not enough information to answer the question.

A

$2.50 of tax liability since the ordinary dividend rate is the taxpayer’s marginal rate.
- Home Depot dividends will qualify for qualified dividend treatment if the individual meets the requisite holding period, which is more than 60 days in the 121 days surrounding the ex-dividend date. Since he only held the stock for 39 days, he does not meet the holding period. Therefore, the tax rate applied against the dividend is his ordinary income tax rate of 25%.

64
Q

Tara is single and her taxable income is $42,750, which puts her in the 22% tax bracket. How much is her income tax liability for 2019?

$952.50
$4,453.50
$5,263.50
$9,405.50

A

$5,263.50
- The calculation is as follows:

Income = $42,750

Taxes = $4,543* + [($42,750 - $39,475*) × 22%] = $5,263.50

*Based on the provided tax tables for 2019 for income below 39,475 plus 22% of any income above $39,475. A copy is located in the front of your pre-study books and in the CFP Board provided tax tables in Phase I.

65
Q

Lucy and Lou are married and normally file a joint return. Under which of the following circumstances are they required to file a tax return?

If Lucy is 64 and Lou is 66 and their gross income is $25,000.
If Lucy and Lou are both 35 and have one dependent and their gross income is $23,000.
If Lucy is 64 and Lou is 64, Lou is blind, and their gross income is $25,000.
None of the above.

A

If Lucy is 64 and Lou is 64, Lou is blind, and their gross income is $25,000.
- The MFJ standard deduction is $24,400 and the additional standard deduction for 65 and over is $1,300.

In option “A”, their total standard deduction is ($24,400 + $1,300) = $25,700. In option “B”, their standard deduction is $24,400. In option “C”, their standard deduction is $24,400. Lou may benefit from an additional $1,300 additional standard deduction for blindness upon filing. A return must be filed to claim the ASD for blindness. The IRS will know your age, but not that you are blind.

66
Q

Kelly and Terry are separated and in the process of divorce so they are going to file their income tax return for last year separately. Kelly made a large number of charitable contributions from her own separate checking account at the end of last year so she wants to file using itemized deductions. What are Terry’s options assuming Kelly files using itemized deductions?

Terry may utilize the standard deduction or itemized deduction, whichever is less.
Terry may utilize the standard deduction or itemized deduction, whichever is greater.
Terry must utilize the standard deduction.
Terry must utilize itemized deductions.

A

Terry must utilize itemized deductions.
- A married individual who files a separate return (married filing separately filing status) cannot use a standard deduction if that person’s spouse itemizes deductions.

67
Q

On January 1st of this year, Linda sold a piece of land she had had for years to George. Linda’s basis in the land was $75,000 and she sold it for $100,000. It was agreed that George would pay Linda $10,000 as a down payment and would make installment payments of $10,000 for the next 9 years plus 10% interest. His second payment was due and payable December 31 of this year. What is Linda’s tax consequence of this transaction this year?

$20,000 of ordinary income
$20,000 of long term capital gain
$2,500 of long term capital gain and $9,000 of ordinary income
$5,000 of long term capital gain and $9,000 of ordinary income

A

$5,000 of long term capital gain and $9,000 of ordinary income
- George is paying her $100,000. Her amount invested is $75,000. Therefore, over 10 years, her total profit will be $25,000 or $2,500 per year except for the down payment. There are two payments at the end of the year.

An interest payment of 10% × $90,000*=$9,000 (ordinary income). The second payment, $10,000, consists of $2,500 capital gain and $7,500 of return of basis.

*Recall the amount paid was $100,000 less a down payment of $10,000, so $90,000 was outstanding.

Total CG RB OI

Down Payment $ 10,000 $ 2,500 $ 7,500 $ -
Dec 31 Payment $ 10,000 $ 2,500 $ 7,500 $ 9,000

68
Q

Veronica has determined that she needs to make a 4th quarter federal estimated income tax payment. When is this payment due?

January 15th of next year.
December 31st of this year.
December 15th of this year.
September 15th of this year.

A

January 15th of next year.

The 4th quarter federal income tax estimated payment is due by January 15th of the year following the year the payment is being made for.

69
Q

Which of the following personal income tax planning techniques are used to defer (postpone) taxation?

I. Itemizing deductions.
II. Contributing to an individual retirement account.
III. Using the child care credit.
IV. Owning cash value life insurance.

a) I and II only.
b) II only.
c) II and IV only.
d) III and IV only.

A

c) II and IV only.
- IRAs and cash value insurance both involve deferral techniques. The other options are avoidance or elimination techniques.

70
Q

Paul (age 35) and his wife Stacey (age 33) are married with three young children. They both work outside the home. Paul is a corporate executive with Wellstar and Stacey is an executive assistant with a small local company. Paul fully participates in his company’s qualified retirement plan by contributing $19,000 of his salary, which is matched 100% up to 3% of compensation. Stacey’s employer does not offer a retirement plan. In addition, during the year they had the following items of income and expense:

Paul’s gross salary: $150,000

Stacey’s gross salary: $32,000

Stacey’s cash gift to her mother: $5,000

Interest from a joint savings account: $100

Federal income taxes withheld from paychecks: $30,000

State income taxes withheld from paychecks: $12,000

Charitable contributions made: $3,400

Mortgage interest for home: $11,100

Real Estate Taxes on home: $6,000

Contribution to Paul’s traditional IRA: $6,000

Contribution to Stacey’s traditional IRA: $6,000

What is Paul and Stacey’s taxable income?

$124,600
$132,600
$134,100
$139,600

A

$132,600
150,000 Paul’s gross salary

+32,000 Stacey’s gross salary

182,000

+ 100 Interest

182,100

-19,000 His 401K

163,600

-6,000 Her IRA (his is not deductible)

157,100 AGI

  • 10,000 State Income Tax Withheld & RE taxes (capped at $10,000)
  • 11,100 Qualified mortgage interest
  • 3,400 Charitable

132,600 Taxable Income

71
Q

Which of the following is a true statement regarding a current Net Operating Loss (NOL)?

A NOL generally can be carried back two years prior to the loss year.
A NOL can only offset up to 80% of the current year’s income.
A NOL deduction is available the year the loss occurs.
An affirmative election must be made to forgo the carryback period.

A

A NOL can only offset up to 80% of the current year’s income.
- NOL losses currently cannot be carried back but they can be carried forward, (except for select agricultural or insurance filers). However, the NOL can only offset 80% of the current year’s income for years after 12/31/17.

72
Q

With regard to Sections 1245 and 1250, Section 1231 will be applied only when:

Any depreciable tangible personal property is sold at a profit.
Any depreciable tangible personal property is sold at a profit above its adjusted (depreciated) basis.
Any depreciable property is sold at a profit above its original cost.
Any depreciable property subject to MACRS rules is sold at a profit.

A

Any depreciable property is sold at a profit above its original cost.
- Section 1231 gain is capital gain. Section 1231 gain occurs when the sale price exceeds the original purchase price.

73
Q

Which of the following creates potential Section 1245?

Amortization of goodwill.
Immediate expense deduction under Section 179.
The sale of tangible personalty used in a trade or business at a gain.
There is Section 1231 loss from the property’s disposition.

A

The sale of tangible personalty used in a trade or business at a gain.
- Depreciable property includes equipment, patents, copyrights and other intangibles. Option “A” is incorrect because goodwill is amortized. Option “B” is incorrect because Section 1245 is not applied to Section 179 expensing. Option “D” is incorrect because a Section 1231 is applied to gains recognized after recapture.

74
Q

In the current year Harold had a Section 1231 gain of $13,000. In the prior years, Harold had the following Section 1231 transactions:

Year	Net Section 1231 Transaction
2018	$4, 000 Loss
2017	$2,000 Loss
2016	None
2015	None
2014	None
2013	$8,000 Gain
2012	$2,000 Gain
2011	        $2,000 Gain
How will Harold's Section 1231 gain be taxed in the current year?

$12,000 will be taxed as ordinary income.
$6,000 will be taxed as ordinary income and $7,000 will be taxed as a Section 1231 capital gain.
$12,000 will be taxed as a Section 1231 capital gain.
None of the above.

A

$6,000 will be taxed as ordinary income and $7,000 will be taxed as a Section 1231 capital gain.
- 5-year Lookback Rule:

Harold would have to recognize $6,000 of his 2019 gain as ordinary income since in 2018 and 2017 he had Section 1231 losses. The remaining $7,000 of Harold’s Section 1231 gain ($13,000 - $6,000) would be treated as a Section 1231 capital gain.

75
Q

A client sold an apartment building last year for $100,000, paying a sales commission of $5,000 plus $2,500 closing costs. The building originally cost $80,000 20 years ago. Total straight line depreciation of $40,000 had been taken. The building had a mortgage of $60,000 which was assumed by the buyer. What is the purchaser’s cost basis?

$70,000
$92,500
$100,000
$107,500
$160,000
A

$107,500
- The cost basis to a purchaser is the acquisition cost plus any other costs associated with purchasing the property or making it useful for service. The buyer paid $100,000. The question indicates that it was the seller and not the buyer who paid the sales commission and the closing costs. Therefore, the buyer’s basis is only the acquisition costs of $100,000. Cash $40,000 plus mortgage $60,000.

76
Q

Donna sells stock in Martin Corporation to her brother David for $1,800. Donna purchased the stock four years ago for $3,000 and the current fair market value of the stock is $1,800. David paid Donna $1,800 for the stock. Which of the following statements is correct regarding the tax consequences of this transaction?

If David subsequently sells the stock to an unrelated party for $3,500, he will realize a gain of $1,700.
Donna has a recognized loss of $1,200.
If David subsequently sells the stock to an unrelated party for $2,200, he will have no gain or loss.
If David subsequently sells the stock to an unrelated party for $3,500, he will have no gain or loss.

A

If David subsequently sells the stock to an unrelated party for $2,200, he will have no gain or loss.
- David will have a double basis in the stock.

77
Q

Tony Scarponi has come to you asking about the basis of property that his brother Calvin gave to him. The property had a market value of $75,000 and Calvin’s adjusted basis in the property was $18,000 at the time of the gift. Calvin paid gift tax of $3,500 on the gift. Tony wants to know what his adjusted basis in the property is. Assume Calvin had utilized his annual gift tax exclusion for gifts previously given to Tony that year. What will you tell him?

Tony’s new basis is $18,000, the same as Calvin’s basis was at the time the gift was made.
Tony’s new basis is the fair market value of the gift at the time of the gift.
The adjusted basis for Tony is $20,660.
The adjusted basis for Tony is $21,500.

A

The adjusted basis for Tony is $20,660.
- Increase in Donee’s Basis = (Appreciation of the Property/ Taxable Gift) × Gift Tax Paid FMV of Property at Date of Gift

[($57,000 ÷ $75,000 = .76) × $3,500] + $18,000 = $20,660

78
Q

Abigail was an original investor in, and owns a 25% interest in, Decorate Your Dream LLC, a home decorating company. Abigail paid $50,000 for her interest. This year, Decorate Your Dream did very well and had a profit of $100,000. However, no distributions were made. What is Abigail’s basis in her interest in Decorate Your Dream at the end of this year?

$25,000.
$50,000.
$75,000.
$100,000.

A

$75,000.
- Abigail’s original cost basis is increased by her share of the profits. Therefore, her original cost basis of $50,000 is increased by $25,000. If there had been a distribution, this would have reduced Abigail’s adjusted basis in her interest in the company.

79
Q

On February 12 of last year, Jason gifted stock in Retro Corp to his brother Aaron. Jason’s basis in Retro’s stock was $12,000 and it had a $9,000 fair market value on the date it was gifted. On August 31 of the following year, Aaron sold the Retro stock to Amy (an unrelated party) for $10,000. What is the tax treatment of these transactions to Aaron and/or Jason?

Jason recognized a loss of $3,000 last year and Aaron recognized a gain of $1,000 in the following year.
Neither Jason nor Aaron has a recognized gain or loss in either year.
Aaron had a recognized gain of $1,000 in the following year.
Jason had a recognized loss of $3,000 last year.

A

Neither Jason nor Aaron has a recognized gain or loss in either year.
- Because of the double basis rule, Aaron’s basis is $12,000 for gain and $9,000 for loss. So he has no gain or loss. Because Jason gifted the shares to a related party, he has no recognizable loss. Double basis simply refers to the two transactions, high and low. The donee uses the FMV of the date of gift ($9,000) as the ‘loss basis,’ and the donor’s adjusted taxable basis ($12,000) as the ‘gain basis.’ When the shares are sold between the FMV (date of gift) $9,000, and the adjusted taxable basis $12,000, the donee uses that value ($10,000) as his adjusted taxable basis.

80
Q

Arthur purchased a home for $175,000. Over the years, Arthur has made the following repairs and improvements: Room addition = $20,000 Pool = $6,000 Roof repair = $3,500 Painting = $2,000 Two years ago, a storm caused damage to the roof. Arthur decided to repair rather than replace the roof. Approximately six months prior to selling the home this year, Arthur had the house painted. What was the basis of the home when Arthur sold it in this past year?

$206,500
$204,500
$201,000
$195,000

A

$201,000
- The basis of the home is increased by the addition of the room and pool. No increase is realized nor recognized at the time of sale for repairs made to the roof or for the painting prior to the sale.

81
Q

Michelle’s husband passed away in January this year. She does not remarry and still maintains a residence for herself and her son who is 10 years old. When she is filing her tax return for this year she may file as:

I. Single
II. Married filing jointly
III. Married filing separately
IV. Qualifying widower

a) I only
b) IV only
c) II and III only
d) II, III, and IV only

A

c) II and III only
- (I) Since Michelle’s spouse died during the year she is considered married for the year. (II) and (III) Since Michelle is considered married for the full year (she was married but her spouse died during the year and she did not remarry) she may file MFS or MFJ. (IV) She does not currently qualify for filing qualifying widower since this status applies for the 2 years following the year of a spouse’s death.

82
Q

Wilma is married to Herb, who abandoned her five years ago. She has not seen or communicated with him since June of that year. She maintains a household in which she and her two young dependent children live. Which of the following statements about Wilma’s filing status in this year is correct?

Wilma can use the rates for single taxpayers for this year.
Wilma can file a joint return with Herb for this year.
Wilma can file as a head of household for this year.
Any of the above.

A

Wilma can file as a head of household for this year.
- Wilma meets the “abandoned spouse” rules. Therefore, she can file as a head of household. Otherwise, her filing status would be married, filing separately. Head of Household required that the taxpayer pay for more than 50% of the upkeep of the home in which the qualifying individuals reside. The qualifying individuals need not be dependents of the taxpayer.

83
Q

Ursula’s divorce from her husband Boris became final on December 30 of the current year. Ursula’s children lived with her for the first four months of the current year, but moved in with their father after Ursula was declared legally blind. Ursula did not contribute anything to the cost of maintaining the household when the children were living with her husband. Ursula is 40 years old. What filing status can Ursula use during the current year and what is her standard deduction?

Married Filing Jointly; $25,700.
Head of Household; $20,000.
Single; $13,850.
Single; $12,200.

A

Single; $13,850.
- Option “A” is incorrect because even though Ursula was married during the current year, she was not married as of the end of the year. Option “B” is incorrect because Ursula does not qualify for the Head of Household filing status. Ursula did not maintain a household for a qualifying child for more than half of the year. Her children only lived with her for four months of the year and she did not pay for the cost of maintaining a household for them for the remainder of the year. Option “C” is correct; Ursula must use the Single filing status. In addition, she is entitled to one additional standard deduction because of her blindness. Therefore, her standard deduction for the current year is $13,850 ($12,200 + $1,650).

84
Q

One of the five tests which must be met to qualify as a dependent is:

The age of the dependent.
The dependent is either a member of the taxpayers household or meets the criteria for family relationship.
The taxpayer is a U.S. citizen.
All of the above.

A

The dependent is either a member of the taxpayers household or meets the criteria for family relationship.
- The five dependency tests are: 1) Gross Income Test, 2) Support Test, 3) Member of Household or Family Member Test, 4) Citizenship Test (U.S., Canada or Mexico), and 5) Joint Filing Test.

85
Q

Which of the following tests must be satisfied by a qualifying child?

I. Relationship Test.
II. Gross Income Test.
III. Abode Test.
IV. Citizenship Test.

a) I and II only.
b) III and IV only.
c) I, II and III only.
d) I, III, and IV only.

A

d) I, III, and IV only.
- The Gross Income Test is a requirement for a qualifying relative, not a qualifying child. All of the other tests must be satisfied by a qualifying child.

86
Q

In which of the following situations, if any, may the individual NOT be deemed a dependent of the taxpayer:

A cousin who does not live with the taxpayer.
A former brother-in-law who does not live with the taxpayer. The taxpayer is divorced.
A nephew who does not live with the taxpayer.
A legally adopted child who does not live with taxpayer.

A

A cousin who does not live with the taxpayer.
- All other parties either satisfy the relationship or member of the household test.

IRS Publication 501: Relatives who don’t have to live with you. A person related to you in any of the following ways doesn’t have to live with you all year as a member of your household to meet this test.

Your child, stepchild, foster child, or a descendant of any of them (for example, your grandchild). (A legally adopted child is considered your child.)

Your brother, sister, half brother, half sister, stepbrother, or stepsister.

Your father, mother, grandparent, or other direct ancestor, but not foster parent. Your stepfather or stepmother.

A son or daughter of your brother or sister. A son or daughter of your half brother or half sister.

A brother or sister of your father or mother.

Your son-in-law, daughter-in-law, father-in-law, mother-in-law, brother-in-law, or sister-in-law.

Any of these relationships that were established by marriage aren’t ended by death or divorce.

87
Q

Eloise is an unmarried elderly woman who lives alone in a small apartment. Eloise is only able to provide 6% of her own support. The remainder of her support is provided by the following people: 10% of her support is provided by her oldest son Frank. 22% of her support is provided by her daughter Gertrude. 30% of her support is provided by her son Henry. 32% of her support is provided by her friend Irene. Which of these individuals is eligible to claim Eloise as a dependent?

Irene can claim Eloise as a dependent because she provides more support than anyone else.
Frank can claim Eloise as a dependent because he is the oldest son.
Henry can claim Eloise as a dependent, but only if Gertrude signs an appropriate statement.
None of these individuals may claim Eloise as a dependent.

A

Henry can claim Eloise as a dependent, but only if Gertrude signs an appropriate statement.
- Option “A” is not correct; Irene may not claim Eloise as a dependent because she does not meet the relationship test as a qualifying relative. Option “B” is not correct; Frank has not provided more than 10% of Eloise’s support, so he is not a qualifying person. Either Gertrude or Henry may claim Eloise as a dependent because they each provided more than 10% of Eloise’s support and together they provided more than 50% of her support. In order for one of them to claim Eloise as a dependent, however, the other must sign a statement agreeing not to claim an exemption for Eloise for this year.

88
Q

Freda purchased a stereo system for her son Wes, age 16. The stereo was placed in Wes’ room and is used exclusively by him. Freda also purchased a new sports car in her own name, that was used 90% of the time by Wes. Which of the cost of these items may be considered as support in determining whether Freda may claim Wes as a dependent?

Both the stereo and the car qualify as support because of the use test.
Neither the stereo nor the car qualify as support because the car is Freda’s and the stereo is diminimus.
The stereo does not qualify for support but the car does because he uses it 90% of the time.
The stereo qualifies for support, but the car does not even though it is de minimus.

A

The stereo qualifies for support, but the car does not even though it is de minimus.
- The stereo system purchased and GIVEN to Wes qualifies as support. Because the car was not GIVEN to Wes (although he is allowed to use it) it will not be considered support. However, maintenance costs, such as gas and insurance that the taxpayer provides for his use of the car will qualify as support.

89
Q

Isaac is a middle school teacher with gross income this year of $35,000. Based on the following, what is Isaac’s adjusted gross income?

  1. $4,000 qualified education interest expense
  2. $2,000 alimony received under a pre-2018 agreement
  3. $1,000 contribution to a traditional IRA

$30,000
$31,500
$32,000
$33,500

A

$31,500
- Isaac’s adjusted gross income is his total gross income of $35,000 - $2,500 in qualified education interest expense (the deductible amount is limited to $2,500) - $1,000 contribution to a traditional IRA = $31,500. The alimony is RECEIVED, which is included in determining his gross income of $35,000. If it was paid then there would be a deduction but that’s not the case.

NOTE: the question gave you his GROSS income, not his salary.

90
Q

Edgar pays alimony under a divorce decree dated 6/1/17 to his former spouse, Frances, in the following amounts: $150,000 in year 1; $40,000 in year 2; and $20,000 in year 3. How much, if any, recaptured alimony must be added to Edgar’s gross income in year 3?

$0.
$32,500.
$112,500.
$150,000.

A

$112,500.
- The alimony recapture in year 3 can be calculated by using the formula: R3 = P1 + P2 - 2P3 - $37,500 R3 is the recapture in year 3. P1 is the payment in year 1, P2 is the payment in year 2, and P3 is the payment in year 3. The amount of alimony recapture in year 3 is $112,500 [$150,000 + $40,000 - (2 × $20,000) - $37,500].

91
Q

Kenny has been a night watchman at Burgundy Company for 10 years. During the current year, he received the following benefits from Burgundy Company: Salary = $15,000; Hospitalization premiums = $3,600; Required lodging on Burgundy’s premises as a condition of Kenny’s employment = $2,400; Reward for preventing a break-in = $1,000. What amount is includible in Kenny’s adjusted gross income for the current year?

$16,000
$17,400
$18,400
$22,000

A

$16,000
- $16,000 ($15,000 salary + $1,000 reward). Hospitalization premiums paid by the employer and any other benefit required by the employer for the employer’s benefit are not included in the employee’s AGI income.

92
Q

Heather had the following transactions for the 2019 tax year, following her 2018 divorce: Alimony received = $2,400; Salary earned = $38,000; Cash dividends received on stock investments = $1,000; Child support payments received = $12,000; Heather’s AGI for this tax year is:

$39,000
$41,400
$51,000
$53,400

A

$41,400
- Child support payments are neither deductible from nor includible in income. Note: alimony received from a contract dated prior to 12/31/18 remains includible income.

93
Q

Paul (age 35) and his wife Stacey (age 33) are married with three young children. They both work outside the home. Paul is a corporate executive with Wellstar and Stacey is an executive assistant with a small local company. Paul fully participates in his company’s qualified retirement plan by contributing $17,500 of his salary, which is matched 100% up to 3% of compensation. Stacey’s employer does not offer a retirement plan. In addition, during the year they had the following items of income and expense:

Paul’s gross salary: $150,000

Stacey’s gross salary: $32,000

Stacey’s cash gift to her mother: $5,000

Interest from a joint savings account: $100

Federal income taxes withheld from paychecks: $30,000

State income taxes withheld from paychecks: $12,000

Charitable contributions made: $3,400

Rent paid for apartment: $24,000

Contribution to Paul’s traditional IRA: $5,500

Contribution to Stacey’s traditional IRA: $5,500

What is Paul and Stacey’s adjusted gross income?

$159,100
$164,600
$176,600
$182,000

A

$159,100
150,000 Paul’s gross salary

+32,000 Stacey’s gross salary

182,000

+ 100 Interest

182,100

-17,500 His 401K

164,600

-5,500 Her IRAs

159,100 AGI

94
Q

Can money paid for child support be structured in a divorce as to be deductible to the payor spouse for divorces prior to 2019?

Yes, if the decree stipulates such.
No, unless the taxpayer gets a letter ruling from the IRS.
No, child support payments can never be made deductible.
Yes, if the money to be considered as child support is included in deductible alimony

A

Yes, if the money to be considered as child support is included in deductible alimony

  • If an agreement is reached (prior to 12/31/18) between former spouses where the decreed amount of alimony is increased to include child support, then the additional alimony would be taxable to the recipient and deductible to the payor. The additional money cannot be based on any contingency such as with the child reaching the age of majority or death.
95
Q

Which of the following is incorrect?

All business deductions are classified as deductions FOR AGI.
Some personal deductions are classified as deductions FROM AGI.
Some business and some personal deductions are classified as deductions FOR AGI.
Some business and some personal deductions are classified as deductions FROM AGI.

A

Some business and some personal deductions are classified as deductions FROM AGI.
- Deductions occur above the line (for) AGI and below the line (from) AGI. All business deductions are for AGI (above the line).

All business expenses (taken by their owners, i.e. sole proprietor or partner) are taken above the line or FOR AGI.

A “business deduction” is tied to the business or owners. These are reported on the sole proprietor’s schedule C, or a partner’s K-1 (which flows onto the Schedule E of Form 1040), for example. Business deductions are all above the line (FOR AGI).

“Personal deductions” are not tied to a business. For example, mortgage interest is a personal deduction below the line (FROM AGI). An HSA contribution is a personal deduction above the line (FOR AGI).

“Job-related employee expenses” used to be deductible as a miscellaneous itemized deductions subject to the 2% floor, a below the line deduction (FROM AGI). Those are no longer available for tax years 2018-2025.

96
Q

Vince, a single individual, is one of the founders and original shareholders of Security Consulting, Inc., a corporate security consulting firm. The company was initially capitalized with $200,000, and Vince was a 50 percent owner. The company was structured as a C corporation and filing requirements and permissible tax elections that could benefit the owners were made at the time the company was created. After several years of successful operations, Security Consulting lost market share to large national firms, and eventually closed down operations. Since it had no assets other than the goodwill of the business, there was nothing left to distribute to the shareholders. Assuming that there were no changes to Vince’s ownership interest over the period of his ownership, and that Vince has no capital transactions in the current year, by how much can Vince reduce his adjusted gross income this year due to the company becoming worthless?

$3,000.
$50,000.
$53,000.
$100,000.

A

$53,000.
-Because it was capitalized with less than $1 million and Vince was an original shareholder, the stock is Section 1244 stock in Vince’s hands. Vince can deduct up to $50,000 of losses as an ordinary loss in any one tax year and the remaining loss is treated as a capital loss. Therefore, Vince will be able to deduct $50,000 of his loss as a Section 1244 loss against ordinary income and will qualify for an additional $3,000 long-term capital loss deduction. The remaining capital loss of $47,000 will be carried forward to future tax years.

97
Q

Child support:

Is deductible when the paying ex-spouse pays the support.
Is deductible when the child or custodial parent uses the money in support of the child.
Is deductible when the decree stipulates the payments will be deductible to the payor and includible to the payee.
Is not deductible.

A

Is not deductible.
- Child support is not deductible under any circumstances when identified as child support. It is also not taxable to the recipient.

98
Q

Billy, single (he was divorced in 2010) and age 42, has the following items of income and expense for the current tax year.

Wages: $60,000

Interest: $1,200

Inheritance: $50,000

Alimony paid: $10,000

Child support paid: $8,000

Federal taxes paid: $5,000

State taxes paid: $2,000

Medical expenses: $7,500

Tickets from his employer for one basketball game: $100

What is his adjusted gross income?

$49,200
$51,200
$51,300
$61,200

A

$51,200
- His income includes the wages and interest and the alimony paid is deductible for AGI. Therefore his AGI = $60,000 + $1,200 - $10,000 = $51,200. Note that the basketball tickets are “occasional” and thus a tax-free fringe benefit.

99
Q

Kenny would like to make a deductible contribution to a Health Savings Account. Which of the following is/are a requirement in order for Kenny to be able to make such a contribution?

I. Kenny must be eligible to establish a Health Savings Account.
II. Kenny must have a high deductible health plan.
III. Kenny must meet the deductible of his HDHP.

I only.
I and II only.
II and III only.
I and III only.

A

I and II only.
- Kenny is not required to meet the deductible of his high deductible health plan in order to make a deductible contribution to his HSA. However, he is required to be eligible to establish an HSA and to have a high deductible health plan.

100
Q

In the current year, Susan and Tom had three preschool-aged children who require daycare so that Susan and Tom work. Their total daycare costs for all three children was $6,500. While at daycare, 1/3 of the time is education while the remainder is custodial care. Assuming that Susan and Tom have an AGI of $110,000, what is the dependent care credit amount?

$1,200
$1,300
$4,800
$6,500

A

$1,200
- Sue and Tom will be able to take a credit of $1,200 ($6,000 × 20%). Although not normally allowable as deductible child care, education expenses for children preschool through kindergarten are qualified expenses. The amount of costs that qualify is the lesser of actual costs or $3,000 for one qualified individual, and $6,000 for two or more qualified individuals (2019). If AGI is great than $43,000 the allowed credit is 20%

101
Q

Which of the following is a tax credit that reduces the tax due on taxable income?

I. Qualified dependent credit.
II. Child tax credit.
III. Earned income credit.
IV. Credit for estimated tax payments.

I, II and III only.
II and III only.
I, II, III and IV only.
III, and IV.

A

I, II, III and IV only.
- The “Qualified dependent credit” is new under TCJA and applies to qualified dependents and/or qualifying children 17 and over. It is limited to $500. The “child tax credit” applies to qualifying children under age 17 and was expanded under TCJA to $2,000 per child, with the possibility of up to $1,400 per child being refundable. The “earned income credit” is a credit against the calculated tax, available to those with very low income, predominantly from earnings (wages) and it is a refundable credit. The CFP exam considers the prepayment of tax, through withholding and/or estimated tax payments, as credits as well since they also reduce the balance due.

102
Q

Which of the following are the types of credits available to individual taxpayers?

I. Refundable.
II. Deductible.
III. Non-refundable.
IV. Partially refundable.
V. Exempted.

I and II only.
I and III only.
II, IV and V only.
I, III and IV only

A

I and III only.
- There are many non-refundable credits including, but not limited to, child care. EIC is the only fully refundable credit. The IRS recognizes two categories of credits; refundable and non-refundable. The term refundable does not necessarily imply that it is fully refundable, it may be fully or partially refundable. AOTC and child tax credit are partially refundable.

103
Q

Karen and Tom are married filing jointly taxpayers with 3 children. Their MAGI is $85,000. What is the maximum amount of the child tax credit that could be refundable to Karen and Tom?

$0.
$2,000.
$4,200.
$6,000.

A

$4,200.
- They have 3 children. The child tax credit is $2,000 per child against their tax obligation, up to $1,400 ($4,200 for the three children) per child can be refundable if there is no tax obligation due.

104
Q

Victor and Vivian have a very diverse family. Which of the following children would not be a qualifying child for the purpose of claiming the child tax credit in the current year?

Vivian’s granddaughter, who turned 4 in the current year and lives with Victor and Vivian for more than half of the year.
Victor’s brother, who turned 16 in the current year and lives with Victor and Vivian for more than half of the year.
Victor and Vivian’s daughter, who turned 17 in the current year and does not provide more than half of her support.
Victor and Vivian’s son, who was born on October 21 of the current year

A

Victor and Vivian’s daughter, who turned 17 in the current year and does not provide more than half of her support.
- Individuals who reach the age of 17 during the tax year are not eligible to be qualifying children for the purpose of the child tax credit. All of the other children are eligible to be qualifying children.

105
Q

The carrybacks and carryforwards associated with the general business credit must be used in a specific order. Which of the following correctly describes that order?

I. The business credit carryforwards to the current year

II. The amount of the current year business credit

III. The business credit carrybacks to the current year

III, II, I
II, III, I
I, III, II
I, II, III

A

I, II, III
- Answer “D” correctly describes the sequence in which the carrybacks and carryforwards associated with the general business credit must be used.

106
Q

Sara’s daughter Tara completed her senior year of college in the current year. Sara paid $5,000 in qualified education expenses for Tara in the current year. Sara is a MFJ taxpayer and has an AGI of $50,000 for the current year. What, if any, education credit will provide Sara the highest credit and how much is that credit?

Sara is not eligible to claim an education credit.
American Opportunity Tax Credit in the amount of $2,500.
Lifetime Learning Credit in the amount of $1,000.
Lifetime Learning Credit in the amount of $2,000

A

American Opportunity Tax Credit in the amount of $2,500.
- Option “B” is correct because the American Opportunity Tax Credit will provide a credit of $2,500 = 100% of the first $2,000 of expenses and 25% of the next $2,000 of expenses. Option “D” is incorrect because Sara would need to have paid $10,000 in qualified education expenses in order to claim the maximum Lifetime Learning Credit of $2,000. Option “C” is incorrect, because although the LLC may provide Sara a credit equal to 20% of her qualified education expenses, or $1,000, she would choose the American Opportunity tax credit - she cannot claim both.

107
Q

Which of the following is not a requirement of the individual real estate investor exception to the passive activity loss rules?

The taxpayer must materially participate in the activity.
The taxpayer must own at least 10% of the value of the real estate.
The taxpayer must have an AGI of less than $150,000.
The taxpayer must actively participate in the activity.

A

The taxpayer must materially participate in the activity.
- The taxpayer is not required to materially participate in the activity, but the taxpayer must actively participate in the activity. Material participation requires substantial, continuous involvement in the operation of the activity. Active participation means that the taxpayer participates in making management decisions concerning the property, but is not substantially and continuously involved in the operation of the activity.

108
Q

Pat invests $150,000 for a 10% interest in a limited partnership. He receives a K-1 with his loss at $80,000. How much of his loss is suspended?

$0.
$8,000.
$15,000.
$80,000.

A

$80,000.
- To determine whether any of the losses are suspended you must first apply the at-risk rules, then the passive loss rules. The amount at risk is the basis of $150,000. Since the loss is less than the amount at risk, none of the loss is suspended due to the at-risk rules. In applying the passive loss rules, the passive loss is limited to the amount of passive income for the year. Since there is no passive income for the year, none of the loss may be recognized and the $80,000 loss is suspended.

109
Q

Which of the following is not an exception to the passive activity rules for rental activities?

If the average period of customer use is seven days or less, the activity could be considered an active trade or business.
If the average period of customer use is 30 days or less but the taxpayer does not provide significant personal services in concert with the rental activity, the activity may be classified as an active trade or business.
The activity will be considered an active trade or business if the rental of property is incidental to the non-rental activity of the taxpayer.
A rental activity that the taxpayer customarily makes available during business hours for nonexclusive use by customers will be classified as the active conduct of a trade or business, provided the taxpayer materially participates.

A

If the average period of customer use is 30 days or less but the taxpayer does not provide significant personal services in concert with the rental activity, the activity may be classified as an active trade or business.
- Option “B” is correct because the taxpayer must provide significant personal services in concert with the rental activity and must materially participate in the activity in order to classify the activity as an active trade or business.

110
Q

Under the First in First Out (FIFO) inventory system:

The first good purchased is the first good sold.
The cost of goods sold is based on the costs of the first goods purchased.
FIFO reduces the probability of scrap or obsolescence.
All of the above.

A

The cost of goods sold is based on the costs of the first goods purchased.
- The FIFO method is concerned with movement of costs through inventory, not goods. The cost of the first units purchased are the first costs to be transferred to cost of goods sold when the goods are sold.

111
Q

Kathy operates a gym. Her customers pay $480 for 48 aerobic classes that can be taken any time in a 24-month period. If Kathy sells one of the agreements to a customer on June 30 of this year, and the customer takes six lessons during the year, how much of the income is Kathy required to recognize as income on this year’s taxes?

$480 if she is a cash or accrual basis taxpayer.
$240 if she is an accrual basis taxpayer.
$60 if she is an accrual basis taxpayer.
$120 is she is a cash basis taxpayer.

A

$60 if she is an accrual basis taxpayer.
- Accrual taxpayers who receive prepaid revenues do not recognize taxable income until the revenue is actually earned. Kathy has earned 6 out of 48 classes or 12.5% (6 / 48) of the revenue collected; 12.5% × $480 = $60.

112
Q

Which of the following statements correctly describes the cash receipts and disbursements method of accounting?

Income is reported as it is earned and expenses are reported as they are incurred.
The cash receipts method is the most difficult accounting method to understand.
A taxpayer who uses the cash method for reporting most items may use a different method for reporting self-employment income.
Reporting of income and expenses is subject to the all events test.

A

A taxpayer who uses the cash method for reporting most items may use a different method for reporting self-employment income.
- Options “A” and ““D” describe the accrual method of accounting, not the cash receipts and disbursements method of accounting. Option “B” is incorrect; the cash receipts method is generally the easiest method of accounting to understand and the simplest to use.

113
Q

Tikia, Inc. is filing their corporate income tax return for last year with a tax-year end of September 29. What is their tax accounting period?

Fiscal
Calendar
Part-year
52-53 week

A

52-53 week
- A fiscal year ends on the last day of a month other than December. A calendar year ends on the last day of December. A partial-year is for a time span less than 1 year. A 52-53 week year ends on a specified day of the week (such as Friday) that occurs in the last week of the last month of the tax year.

114
Q

Under the accrual method of accounting, the taxpayer (seller) recognizes income when:

The bill is received by the buyer.
The goods are accepted by the buyer.
The goods are loaded on the truck at the seller’s facility.
The seller writes and sends the invoice after sending the goods.

A

The seller writes and sends the invoice after sending the goods.
- The accrual accounting method recognizes income when the taxpayer has a right to collect. This occurs usually after the completion of a job and in no case later than when the invoice is prepared and sent.

115
Q

The accrual method of accounting generally must be used to report income earned by:

A C corporation that made a S election.
A general partnership with a corporate partner.
A LLC that has business in most or all states.
None of the above.

A

A general partnership with a corporate partner.
- The key is the question of when the accrual system “Must” be applied. C Corporations must use the accrual method. Therefore, any entity where a C corporation is a partner or owner must use the accrual method. The correct answer is “B” - Partnerships with corporations as owners must use the accrual method. Option “D” is automatically eliminated because S corporations are not required to use the accrual method of accounting.

116
Q

Under what circumstances is a taxpayer required to use a calendar year tax period?

If the taxpayer does not keep books or accounting records.
If the taxpayer opens a new business.
If the taxpayer has an initial tax year of less than 12 months.
If the taxpayer receives reporting documents such as Forms W-2 and 1099.

A

If the taxpayer does not keep books or accounting records.
- Option “B” is incorrect because there is no requirement for new businesses to use a calendar year tax period. Option “C” is incorrect; a taxpayer may use a fiscal year tax period and have a tax year of less than 12 months in the first year. Option “D” is incorrect; although most taxpayers who receive such documents use the calendar year tax period, the receipt of such documents does not in and of itself require them to do so.

117
Q

Blake is a CFP® professional and prepares tax returns for his clients. He prepared his brother’s income tax return for $1,000 and he willfully neglects to include $30,000 of income since his brother did not receive a 1099 for consulting work. Blake is aware that his brother earned the $30,000 but fails to report it since he doesn’t believe the IRS will catch the understatement of income. The additional tax on this $30,000 of income would have been $7,500. How much of a penalty may Blake be subject to for the understatement of income?

None, but his brother will be subject to penalties.
$3,750
$5,000
$7,500

A

$5,000
- The preparer penalty for willful or reckless conduct is the greater of $5,000 or 50% of the income derived by the preparer for the return. In this case, he charged his brother $1,000.

118
Q

Ford’s federal income tax return was due on April 15 of the current year, but Ford did not file his return or pay his taxes until June 30 of the current year. Ford’s unpaid tax balance during this period was $400. What is the total penalty that will be imposed on Ford for his failure to file and failure to pay?

$60.
$66.
$215.
$209.

A

$215.
- The failure to file penalty is 5% of the unpaid tax balance for each month or part thereof that the tax return is late (up to 25% of the unpaid tax balance). Therefore, Ford’s failure to file penalty is $60 (3 months × $400 × 5%). However, if a tax return is filed more than 60 days late (as it is in Ford’s case), the minimum failure to file penalty is the lower of $215 or 100% of the tax due.

  • Therefore, Ford’s failure to file penalty is actually $215. Ford is also subject to a failure to pay penalty of 0.5% per month or part thereof. Therefore, Ford’s failure to pay penalty is $6 (3 months × $400 × 0.5%). Note that the failure to file penalty is reduced by the failure to pay penalty. Therefore, Ford’s total penalty is $215 ($215 failure to file penalty - failure to pay= $209, plus $6 failure to pay penalty).
119
Q

Under which of the following circumstances will a taxpayer be subject to an accuracy-related penalty?

I. If the taxpayer files an incorrect return and has failed to make a good faith effort to comply with the tax law.
II. If the taxpayer understates his tax liability by more than 5 percent of the correct tax liability.
III. If the taxpayer makes a substantial understatement associated with an estate or gift tax valuation.

I only.
I and II only.
II and III only.
I and III only.

A

I and III only.
- A taxpayer will be subject to an accuracy-related penalty if he makes a substantial understatement of his tax liability, generally more than 10 percent of the correct tax liability and at least a $5,000 tax deficiency.

120
Q

Mr. Rangel files a fraudulent income tax return (again) and has a $10,000 income tax deficiency because of it. What is the amount of his penalty?

$10,000
$8,000
$7,500
$5,000

A

$7,500

- The penalty for filing a fraudulent income tax return is 75% of the deficiency.

121
Q

Which of the following qualify for the medical expense deduction?

Maternity clothes for a celebrity.
The cost of non-prescribed insulin.
A weight reduction program designed to avoid diabetes.
A breast implant for an aging starlet.

A

The cost of non-prescribed insulin.
- Non-prescribed medicine is generally not deductible, but an exception to this is insulin. Prescribed or non-prescribed insulin is considered a deductible medical expense. All others including clothing, weight loss programs or elective surgery are not deductible medical expenses but A and D could be business expense deductions.

122
Q

Veronica borrowed $300,000 to acquire a parcel of land to be held for investment purposes. During the year, she paid interest of $30,000 on the loan. She had AGI of $70,000 for the year. Other items related to Veronica’s investments include the following:

Investment income = $15,200

Long-term gain on the sale of stock = $6,000

Investment counsel fees = $900

Veronica is unmarried and elected to itemize deductions. She had no miscellaneous deductions other than the investment counsel fees. Determine Veronica’s maximum investment interest deduction.

$30,000
$21,200
$16,100
$15,200

A

$21,200
- The taxpayer’s investment interest deduction is limited to the investment income. The investment income is $15,200 plus she can add the capital gains of $6,000 and deduct $21,200. The excess investment interest ($30,000 - $21,200) can be carried over to next year.

123
Q

Janice had a car accident in which the other car’s driver was cited for reckless driving. An appraisal established that the fair market value of Janice’s car declined by $4,000 as a result of the accident. To have the car repaired, Janice paid $5,000. Her insurance company reimbursed her $500. If Janice’s AGI for the year was $20,000, determine her deductible casualty loss on the car.

$0
$1,400
$1,500
$2,400

A

$0

- Casualty loss expense for non-business losses are no longer deductible after 12/31/17.

124
Q

Mel made the following contributions to charity during the past year:

Used clothing of the taxpayer and family - Basis = $900 and FMV = $300

Stock in GMC held as an investment for 13 months - Basis = $8,000 and FMV = $7,000

Stock in United Corp. held as an investment for 9 months - Basis = $9,000 and FMV = $10,000

Real estate held as an investment for six years - Basis = $10,000 and FMV = $25,000

The used clothing was donated to the Salvation Army; the other items of property were donated to a Methodist seminary. Disregarding any percentage limitations, Mel’s charitable contribution deduction is:

$43,300
$42,900
$41,300
$26,300

A

$41,300
- A donation of short-term capital assets is recognized as a charitable expense at the lower of the FMV or basis. Donations of appreciated long-term assets are recognized as a charitable expense at the fair market value unless basis is chosen. In this question, the United Corp stock is a short-term investment that was held for less than 12 months, therefore, it will be expended as a charitable contribution at the donor’s basis of $9,000. The remainder of the donated assets are expensed for charitable purposes at their fair market value.

125
Q

Lauren has purchased a home worth $1.5 million with an interest-only mortgage of $1.2 million on 12/20/17. She is currently only paying interest on the mortgage in the amount of $60,000 per year. What amount may she deduct as home mortgage interest on Schedule A of her individual income tax return?

$0
$37,500
$55,000
$60,000

A

$37,500
- The calculation is calculated by dividing the qualified mortgage over the total mortgage times the interest paid.

(750,000/1,200,000) × 60,000 = 37,500

  • Per TCJA, home mortgages are limited to qualified residential interest and a maximum indebtedness of $750,000 if financed after 12/15/17. (For debt prior to 12/15/17, the $1 million limit applies.)
126
Q

Which of the following statements concerning hobby activities is correct?

Any activity which does not generate a profit within three years must be treated for income tax purposes as a hobby activity.
The IRS must prove that the taxpayer does not have a profit motive to treat an activity as a hobby activity.
Expenses associated with the hobby activity can offset, without limitation, the income generated from the activity.
Hobby income is included in gross income above the line, while hobby expenses are not deductible.

A

Hobby income is included in gross income above the line, while hobby expenses are not deductible.
- Income generated from a hobby activity is included in gross income, and expenses associated with the hobby are no longer deductible per TCJA for years after 12/31/17.

  • If the activity earns a profit in three out of five years, the IRS has the burden of proof of showing that there is no profit motive, but if there has not been a profit in three out of the last five years, the taxpayer has the burden of proof. There is no bright line test that requires an activity to be treated as a hobby activity based on the income trend of an activity.
127
Q

In September of this year, Rudolph refinanced his home. Prior to refinancing, his only outstanding debt was the balance due on his original mortgage of $110,000. Rudolph needed some additional money to pay for his child’s college education and to take advantage of an investment opportunity, so upon refinancing, Rudolph took out a 30-year mortgage for $250,000. To reduce the interest rate on the mortgage, down to 5%, Rudolph paid $2,500 in points to refinance. Which of the following statements is correct?

a) Rudolph can deduct all of the mortgage interest paid on the note.
b) Rudolph can deduct the mortgage interest incurred on $110,000. The remaining interest is not deductible.
c) Rudolph can only deduct the mortgage interest incurred on $210,000.
d) Rudolph can only deduct the mortgage interest paid on $210,000 plus the points paid on refinancing (ratably over the life of the loan).

A

Rudolph can deduct the mortgage interest incurred on $110,000. The remaining interest is not deductible.
- $110,000 of the refinanced amount continues to be treated as acquisition indebtedness since that was the previous balance of Rudolph’s mortgage. An additional $140,000 will be considered home equity indebtedness but is not deductible based on its use. Anytime a pre-TCJA mortgage is refinanced for more than the existing balance, the mortgage will follow the current TCJA rules.

  • C and D represents the amount under the pre-TCJA rules; up to 100,000 of home equity could be deducted in addition to the mortgage.
  • Under TCJA only qualified residential interest (acquisition indebtedness plus home improvement portions of additional loans) may be deducted and that is capped at $1 million of debt ($750,000 for contracts after 12/15/17).
128
Q

Billy, single (he was divorced in 2010) and age 42, has the following items of income and expense for the current tax year.

Wages: $60,000

Interest: $1,200

Inheritance: $50,000

Alimony paid: $10,000

Child support paid: $8,000

Federal taxes paid: $5,000

State income taxes paid: $2,000

Medical expenses: $7,500

Tickets from his employer for one basketball game: $100

What is the medical expense deduction that will actually be utilized for the current year?

$0
$2,400
$3,675
$7,500

A
$0
Wages	$60,000
Interest	$1,200
less Alimony	
AGI	$51,200
Medical expenses = $7,500 - ($51,200 × 10%) = $2,380

Medical expenses + taxes paid ($2,380 + $2,000) is less than $12,200 therefore, you will use the standard deduction.

129
Q

Frank’s automobile, which is used exclusively in his business, was damaged in an accident. The adjusted basis prior to the accident was $8,000. The fair market value before the accident was $10,000 and the fair market value after the accident is $500. Insurance proceeds of $10,000 less his $2,000 deductible are received by Frank. What can Frank deduct as a result of the accident?

$0
$1,500
$8,000
$9,500

A

$0
- Frank is not entitled to any “deduction” since the net insurance proceeds were equal to his adjusted taxable basis.

For the tax years 2018 - 2026, deductions for personal casualty and theft losses are suspended by the TCJA of 2017. Business deductions for casualty and theft have remained (to the extent they exceed insurance reimbursement).

130
Q

Which of the following is a deductible miscellaneous itemized deductions?

Union dues.
Appraisal fees for establishing a casualty loss.
Job hunting expenses.
Gambling losses to the extent of gambling gains.

A

Gambling losses to the extent of gambling gains.
- Gambling losses and all deductions incurred in carrying out wagering transactions are limited to gambling winnings for 2018 - 2025. TCJA of 20177 suspended itemized deductions that are subject to the 2% floor have been suspended after Dec 31, 2017 - before Jan 1 2026.

131
Q

Mortimer is an avid collector of antiques associated with the funeral industry. The local hospital is running a campaign to redecorate and expand their lobby, and as a show of support, Mortimer donates a 19th century horse-drawn hearse in mint condition to the hospital. He purchased several of these hearses 30 years ago for $300 each, but the current estimated market value of the hearse today is in the range of $30,000. The hospital decides to sell the hearse and dedicate the proceeds to the renovation effort. Mortimer’s AGI is $50,000. Which of the following statements concerning Mortimer’s charitable deduction is correct?

Mortimer’s deduction for the current tax year will be limited to $25,000.
Mortimer’s deduction for the current tax year will be limited to $15,000.
At least $5,000 of the deductible amount will have to be carried over to future tax years.
Mortimer’s income tax deduction is $300.

A

Mortimer’s income tax deduction is $300.
- When tangible personal property donated to a charity will not be used by the charity to carry out its tax-exempt purpose, the deduction available to the donor is limited to the donor’s cost basis and will be subject to the 50 percent limitation. Redecorating the lobby is not part of the hospital’s tax-exempt purpose. In this case, Mortimer’s cost basis is $300 and since 50% of his AGI is $25,000, Mortimer may take his entire charitable deduction this year.

132
Q

Matt owned stock in Whitline Corporation that he donated to a university (a qualified charitable organization) on May 1 of this year. What is the amount of Matt’s charitable income tax deduction assuming that he had purchased the stock for $3,500 on March 3 of this same year, and the stock had a value of $4,600 when he made the donation? His AGI was $8,100.

$1,100
$3,500
$4,050
$4,600

A

$3,500
- Taxpayers who donate short term gain property are required to use the lesser of fair market value or adjusted basis for the determination of the charitable income tax deduction. In this case, Matt only owned the stock for 2 months, therefore it will be short term. The basis ($3,500) in less than the FMV ($4,600).

133
Q

Nathan sustained serious facial damage in a snow skiing accident. To restore his physical appearance, he had restorative surgery. At the same time, he also had surgery to remove wrinkles from under his eyes caused by age. Which of these procedures can Nathan claim as a deductibe medical expense?

Both of the procedures can be claimed as deductible medical expenses because he had them done at the same time.
Only the restorative surgery can be claimed as medical expenses.
Only the wrinkle removal surgery can be claimed as medical expenses.
None of these can be claimed as medical expenses because they are both cosmetic.

A

Only the restorative surgery can be claimed as medical expenses.
- Reconstructive surgery to correct disfigurement is a deductible medical expense subject to 10% AGI. Elective cosmetic surgery is not deductible

134
Q

Elaine incurred $26,000 of margin interest on her $600,000 investment portfolio. Her portfolio income consists of $10,000 in interest, $15,000 in qualified dividends, $3,000 in ordinary dividends, $6,000 in short-term capital gains, and $11,000 in long-term capital gains. How much of the margin interest is deductible on Elaine’s tax return assuming no special elections?

$17,000
$19,000
$24,000
$26,000

A

$19,000
- She can deduct up to her net investment income which = $10,000 + 3,000 + 6,000 = $19,000 without making a special election. If she elected to treat the long-term capital gains and qualified dividends as ordinary income she could deduct it all. However, DO NOT assume that the election is made. The question would have to specify that information or ask “what is the maximum she can deduct?”

135
Q

Ima Clipper, a well-known artist, donated one of her original bronze creations to a local charity, which auctioned the piece for $3,000. Ima totaled her costs as follows:

Bronze = $425

Other materials = $150

Pro-rata overhead = $125

Furnace/casting fees = $200

Artistic contribution = $2,100

Assuming this is Ima’s only charitable contribution and based on an annual income of $150,000, what is the maximum amount of charitable income tax deduction available to her?

$3,000
$2,100
$775
$900

A

$775
- Only materials and expenses are deductible, not artistic contribution or time. No deduction is allowed for use of property; therefore, the pro-rata overhead would likely not be allowed.

136
Q

During the current year, Justin, a self-employed individual, paid the following amounts: Federal income tax = $5,000; State income tax = $3,000; Real estate taxes on land in a neighboring state (held as an investment) = $8,000; State sales taxes = $600; State occupational license fee = $400. What amount can Justin deduct as taxes by itemizing his deductions?

$3,600
$10,000
$11,000
$11,600

A

$10,000
- State income or sales tax (whichever is higher) and real estate taxes are deductible as itemized deductions but for years after 12/31/17, they are capped at $10,000 (total). Federal income taxes paid are deductible from the tax liability, but are not an itemized deduction. Occupational license fees are deductible as a direct business cost in which they are deducted for AGI not as itemized deductions.

137
Q

Olive’s daughter Polly suffers from a rare illness. During the current year, Olive drove Polly to see a specialist in another state 15 times. Each trip was 300 miles each way and required an overnight stay in a hotel that costs $70 per night. Olive’s AGI is $24,000. What is her medical expense deduction for 2019?

$2,400
$450
$300
$0

A

$450
- Olive may deduct $0.20 (2019) cents per mile for the travel associated with Polly’s medical care and may deduct up to $50 per night per person for lodging. Therefore, the total medical expenses are $2,850 which is calculated as follows: [(300 x 2 x 15 x $0.20=1,800) + (15 x $70=1,050)]. The trip mileage is stated as 300 miles each way, you will need to multiply by 2 and use round trip mileage.

  • However, Olive may only deduct the amount that exceeds 10% of her AGI. 10% of Olive’s AGI is $2,400. Therefore, Olive’s medical expenses deduction is $450. Note, the full $70 is allowed because the rule is $50 per person per night.
138
Q

Franco and Giada are trying to calculate their gross income for the current year. Which of the following items should they exclude from their gross income?

I. $60,000 in cash inherited by Giada from her mother.
II. $20,000 borrowed by Franco and Giada from First City Bank.
III. $12,000 gain from the sale of Franco and Giada’s boat.
IV. $400 of interest earned on a loan made by Franco to his cousin Vinnie.

a) I and II only.
b) III and IV only.
c) I, II and III only.
d) I, II and IV only.

A

a) I and II only.
- Inherited cash or property is excluded from gross income; therefore, option “I” is correct. Borrowed money is also excluded from gross income; therefore, option “II” is also correct. Gain on the sale of assets (option “III”) and interest income (option “IV”) are both included in gross income.

139
Q

Bill and Renee are married and in community property but living apart and filing separate federal income tax returns. Each earned a salary of $25,000 and Renee received $5,000 in interest on money she inherited from her deceased mother after her marriage to Bill. Which of the following is correct?

In some states, Bill’s gross income is $55,000.
In some states, Renee’s gross income is $27,500.
In all community property states, Bill’s gross income is $25,000.
In all community property states, Renee’s gross income is $30,000.

A

In some states, Renee’s gross income is $27,500.
- The inherited property may be considered separate property, and therefore, in some community property states the income earned from it is separate. In others, it is community.

140
Q

James received an academic scholarship to State University. He is a candidate for a degree. Under the scholarship agreement he received:

Tuition ($1,500)
Books ($400)
Room and board ($5,000)
What is James’ gross income, if any, from the scholarship?

None
$5,000
$5,400
$6,900

A

$5,000
- James’ gross income is $5,000. Tuition and books are not included in income, but living expenses such as room and board are included in taxable income.

141
Q

Aurora had the following cash inflows during the current taxable year:

Wages: $45,000
Loan Proceeds: $2,000
Child Support: $5,000
Stock Sale Proceeds: $3,000
U.S. Government Bond Interest: $1,000
What is her gross income for income tax purposes if her adjusted tax basis in the stock was $2,000?

$45,000
$47,000
$49,000
$51,000

A

$47,000
- Wages plus stock sale proceeds less basis in the stock plus U.S. government bond interest = $45,000 + $3,000 - $2,000 + $1,000 = $47,000. Loan proceeds are not taxable income and neither is child support.

142
Q

Ian, a single taxpayer, received $15,000 of Social Security retirement benefits in the current year. He also received $16,000 of interest income. How much of Ian’s Social Security benefits must be included in his gross income?

$0.
$7,500.
$12,750.
$15,000.

A

$0.
- Since the total of Ian’s MAGI ($16,000) and one-half of his Social Security benefits (0.50 × $15,000 = $7,500) is less than the base amount ($25,000), none of his Social Security benefits are included in gross income.

143
Q

Punitive damages received as a result of personal injury:

Are not taxable because the damages are intended to punish the other party.
Will be non-taxable if the payments were received on account of a physical personal injury.
Are taxable.
Are not taxable if the personal injury was to one’s business reputation.

A

Are taxable.
- Punitive damages are always included in income. They may be excludible if they are the only recovery in a wrongful death case (The Alabama Rule).

144
Q

After 1989, Donna purchased series EE savings bonds for $2,500 at the age of 25. This year she redeemed the bonds for $5,000 and paid qualified higher education expenses for her daughter in the amount of $3,000. How much interest will Donna be required to include in her gross income this year?

$0.
$1,000.
$1,500.
$2,500.

A

$1,000.
- Because Donna did not use all of the proceeds from the bond redemption to pay for qualified education expenses, she will be required to include part of the interest income from the bonds in her gross income. Donna may exclude $1,500 of interest income from her gross income [($3,000 / $5,000) × $2,500]. Therefore, Donna must include $1,000 in her gross income ($2,500 - $1,500).

145
Q

Edward told his nephew that if the nephew would care for Edward in his old age, the nephew could have all of Edward’s securities when he died. At the time of the promise, the securities had a fair market value of $50,000. The nephew took good care of Edward, whose will left the securities to the nephew. The fair market value of the securities at the time of Edward’s death was $80,000. Edward could have gone to a nursing home and obtained the same services as provided by the nephew for $40,000. The nephew’s gross income from the above is:

$0; this is an inheritance.
$40,000; this is earned income at the fair market value.
$50,000; this is earned income as of the time of the promise.
$80,000; this is earned income equal to the date of death value.

A

$80,000; this is earned income equal to the date of death value.
- Because the agreement was to compensate Edward for his services, even though the transfer occurred following death, it is not a gift or bequest. It is compensation for services performed. Compensation of property has a value equal to its fair market value on the date of transfer. The fact that Edward died and a step up in basis would ordinarily occur is immaterial.

146
Q

Tom operates an illegal drug operation and incurred the following expenses: Salaries = $50,000; Illegal kickbacks = $20,000; Bribes to border guards = $25,000; Cost of goods sold = $150,000; Rent = $8,000; Interest = $10,000; Taxable income = $400,000. How much is his taxable income reduced by, based on the above expenses?

$-0-
$150,000
$218,000
$263,000

A

$150,000

- Cost of goods sold is the only deduction allowed for illegal drug operation activities.

147
Q

Last year, Monique took a trip from Boston to Rome. She was away from home for 15 days. She spent five days vacationing and ten days on business for her clothing line (including the two travel days.) Her expenses are as follows: Airfare = $2,100; Lodging (15 days) = $3,150; Meals (15 days) = $2,400; Valet service (cleaning of business suits) = $60. Monique’s business travel expenses deduction is:

$4,360
$4,660
$5,060
$5,860

A

$4,360
- She owns her business and can take a business deduction. If she was an employee of the company it would be zero under the TCJA suspension of itemized deduction subject to the 2% floor.

Airfare Deductibility Rules:

DOMESTIC
If primarily business then deduct all airfare.
Prorata meals and lodging.

FOREIGN 
Prorata meals and lodging.
Prorata airfare unless (then you can deduct all):
< 7 days
<25% on personal
No control
Vacation not a deciding factor
Airfare = $2,100 × (10 ÷ 15) = $1,400
Lodging = $3,150 × (10 ÷ 15) = $2,100
Meals = $2,400 × (10 ÷ 15) = $1,600 × 50% (meals are only 50% deductible) = $800
Valet = $60
Total = $4,360
148
Q

Albacore, Inc., an accrual method taxpayer, was incorporated on January 2 this year but did not begin business operations until July 1. Albacore adopted a calendar year tax year and incurred the following expenses during its first tax year:

Incorporation fees paid to State: $150

Expenses in connection with issuing and selling stock: $1,800

Legal fees incident to incorporation: $1,650

If Albacore, Inc. makes an appropriate and timely election, the maximum organizational expenditures that it can properly deduct for the current year would be:

None
$1,800
$3,450
$3,600

A

$1,800
- Expenses incurred in connection with issuing and selling stock are not deductible. The rule is the lesser of expenditures or $5,000. Therefore, $1,650 + 150 = $1,800.

149
Q

Which of the following must be capitalized by a business?

A major tune-up of a truck used in the business.
The replacement of a windshield of a business truck, which was broken in an accident.
The repair of a roof damaged by a hurricane.
The cost of a concrete pad upon which to place a machine.

A

The cost of a concrete pad upon which to place a machine.
- Costs to improve, better, or extend the life of an asset are capitalized. Tune-ups are a maintenance activity. Maintenance and repairs (roof and windshield) are period costs deductible in the period in which they were incurred.

150
Q

Which of the following is not a requirement for a deductible business-related expense?

The expense must be ordinary.
The expense must be necessary.
The expense must be capitalized.
The expense must be reasonable.

A

The expense must be capitalized.
- Deductible business expenses are usually not capital in nature. Capitalized expenses are recovered over the life of the asset through depreciation deductions, rather than through a deduction for a business expense.

151
Q

Which of the following is a “trade or business” expense?

QNEC contribution to 401K plans to retain qualification.
Charitable contributions by a partnership.
Gifts to contract letting officals to get preferential treatment in contracts.
Prepaid parking fines paid at a discount to the city to avoid ticketing in high traffic areas.

A

QNEC contribution to 401K plans to retain qualification.
- Charitable contributions by a partnership are personal expenses. Fines are not deductible. Gifts (bribes) are not deductible. QNEC, qualified non-elective contributions, to a 401K plan to maintain qualification is an ordinary/necessary business expense.

152
Q

Earl went from Portland, Oregon to New York City on business. After six days of business meetings promoting his new winery in hopes of expanding distribution, he took four days of vacation to go sightseeing. Earl’s expenses for the trip are as follows:

Airfare = $1,200

Lodging (10 days × $90) = $900

Meals (10 days × $100) = $1,000

Airport limo = $60

Earl’s business deduction is:

$2,400
$2,100
$1,620
$1,896

A

$2,100
- The expenses associated with a trip that are for non-business purposes are not deductible from income. Remember that meals are only 50% deductible. Accordingly, the answer is calculated as follows: $1,200 (airfare) + $60 (limo) + $540 (hotel - 6 days at 90/day - the remaining 4 days are personal and not deductible) + $300 (meals - 6 days at 100/day × 50% meal allowance) = $2,100. The airfare (to and from) was transportation for business that would not change whether it was personal or not, so it is not pro-rated but rather fully deductible. $1,200 + $60 + $540 + $300 = $2,100. Airfare Deductibility Rules:

  • Domestic: If primarily business then deduct all airfare. Prorata meals and lodging.
  • Foreign: Prorata meals and lodging. Prorata airfare unless (then you can deduct all): < 7 days <25% on personal

No control; vacation not a deciding factor

153
Q

Alice owns land “A” with an adjusted basis of $250,000, subject to a mortgage of $50,000. On July 1st, Alice exchanges land “A” and its mortgage for $300,000 in cash, a promissory note for $300,000, and property “B” that has a fair market value of $75,000 with Betty. What is the amount realized by Alice?

$675,000
$725,000
$925,000
$975,000

A

$725,000
- The realized amount not only includes the monies and fair market value of property “B” received (and any indebtedness the buyer has to the seller), but also any liabilities for the seller is relieved. In this case, the seller received $675,000 in cash, property, and notes (buyers indebtedness to the seller) as well as relief from $50,000 in mortgage. The total amount realized is $725,000

154
Q

John owns a rental home in Arizona. He decided that he would like to acquire a rental home in Washington. Ted who lives in Washington has a rental home. For health purposes, Ted must relocate to Arizona. John and Ted decide to exchange properties under section 1031 of the code. The other facts pertaining to the exchange are: Ted’s Basis = $100,000 John’s Basis = $75,000 Ted and John exchange the two properties, but Ted has to give John an additional $25,000 in cash. The fair market value of Ted’s property is $100,000, and the fair market value of John’s property is $125,000. What is John’s basis in the property received in the exchange?

$75,000
$50,000
$25,000
$100,000

A

$75,000
- Since John received boot and “traded down” his recognized gain equals the lesser of the realized gain or boot received, which is $25,000. John’s basis will be his original basis ($75,000) less the boot received ($25,000) plus the gain recognized ($25,000) or $75,000. Ted’s basis will increase by $25,000 because he is paying $25,000 in boot.

                    John	Ted FMV	               125	100 Basis	                75	100 Deferred Gain	50	0

Boot 25 0
Recognized 25 0
Deferred 25 0

New Asset 100 125
New Basis 75 125
Deferred Gain 25 0

155
Q

Frank is considering selling a parcel of raw land located in South Dakota that he owns. If Frank sells the land, he would like to invest the proceeds in another piece of real property and would like to qualify for like-kind exchange treatment. Which of the following assets would not qualify as like-kind property for the sale of raw land?

Raw land located in Florida.
Raw land located in Canada.
New land located in South Dakota.
An industrial warehouse located in California.

A

Raw land located in Canada.
- U.S. real estate and foreign real estate are not like-kind assets for income tax purposes. Therefore, if Frank exchanges his raw land in South Dakota for raw land in Canada, he will not qualify for like-kind exchange treatment.

156
Q

Peyton has a warehouse used in his business. He exchanges it for a like-kind asset owned by Eli. (Peyton and Eli are unrelated). The basis of Peyton’s asset is $40,000 and he gives Eli $20,000 cash plus the asset in exchange for Eli’s asset, which is worth $36,000. Eli’s basis in his original asset is $10,000. What is Peyton’s new basis?

$20,000
$40,000
$60,000
$80,000

A

$60,000
- Peyton

Before Exchange After Exchange
(Old Property) (New Property)
FMV $16,000 FMV $36,000
Basis 40,000 New Basis 60,000
Potential Loss $24,000 Potential Loss $24,000

157
Q

Peyton has a piece of equipment used in his business. He exchanges it for a like-kind asset owned by Eli. (Peyton and Eli are unrelated). The basis of Peyton’s asset is $40,000 and he gives Eli $20,000 cash plus the asset in exchange for Eli’s asset, which is worth $36,000. Eli’s basis in his original asset is $10,000. What is Peyton’s recognized gain or loss?

$0
$4,000 loss
$24,000 loss
$6,000 gain

A

$24,000 loss
- While losses are not recognized in like kind exchanges; this is not a like kind exchange. Only property is eligible to like kind treatment under the new law. As a result, Peyton has a realized/recognized loss as follows. Value received (new property) $36,000 - Basis $40,000 - cash $20,000 = $24,000 loss. His new basis is the value he “paid” $36,000.

158
Q

Which of the following is not a requirement for the deferral of gain in a nonsimultaneous exchange under Section 1031?

The replacement property must be like-kind property with respect to the original property.
The proceeds from the sale of the original property must be held by an escrow agent.
A replacement property must be identified within 90 days of the sale of the original property.
The closing on the replacement property must take place by the earlier of 180 days from the sale of the original property or the due date (including extensions) of the tax return for the year the original property was sold.

A

A replacement property must be identified within 90 days of the sale of the original property.
- A replacement property must be identified within 45 days, not 90 days, of the sale of the original property.

159
Q

Which of the following is true of the substitute basis of a qualifying asset in a like-kind exchange?

The substitute basis is the asset’s fair market value increased by the gain realized but not recognized.
The substitute basis is the asset’s basis reduced by the gain realized but not recognized.
The substitute basis is the asset’s basis increased by the gain realized but not recognized.
The substitute basis is the asset’s fair market value reduced by the gain realized but not recognized.

A

The substitute basis is the asset’s fair market value reduced by the gain realized but not recognized.
- Substitute basis is the fair market value of an asset, reduced by gain realized, but not recognized.

160
Q

Peyton has a warehouse used in his business. He exchanges it for a storage building owned by Eli. (Peyton and Eli are unrelated). The basis of Peyton’s asset is $40,000 and he gives Eli $20,000 cash plus the asset in exchange for Eli’s asset, which is worth $36,000. Eli’s basis in his original asset is $10,000. What is Eli’s new basis?

$0
$10,000
$20,000
$30,000

A
$10,000
- Eli	 	 	 	 
Before Exchange	 	 	After Exchange	 
(Old Property)	 	 	  (New Property)	 
FMV	$36,000	 	  FMV	$16,000
Basis	10,000	 	  Carryover Basis 10,000
Potential Gain  $26,000  Potential Gain $6,000
161
Q

On December 1, Andrea reviews her investment portfolio and finds out that she has had a very profitable year. To offset some of her gains, Andrea sells 100 shares of Big Bear Corporation for $10,000. She purchased those shares for $15,000 two years earlier. On December 25 of the same year, Andrea reads a newspaper article indicating that the price of Big Bear Corporation is expected to increase substantially. Second-guessing the wisdom of selling her previous shares of Big Bear stock, she purchases 100 shares of Big Bear Corporation for $8,000. What are the tax consequences to Andrea this year?

A $5,000 realized, but not recognized loss.
An $8,000 realized and recognized loss.
A $5,000 realized and recognized loss.
A $7,000 realized, but not recognized loss.

A

A $5,000 realized, but not recognized loss.
- Since Andrea purchased and sold substantially identical securities within 30 days, a wash sale occurs. Her realized loss on the sale of the original shares is calculated as follows: Amount Realized: $10,000 Less: Adjusted Basis -$15,000 Equals: Gain or (Loss) ($5,000) Due to the wash sale transaction, however, Andrea will not be permitted to recognize the loss in the year it was incurred. Instead, the realized but unrecognized loss of $5,000 will be added to the basis of the replacement securities. Andrea purchased the replacement securities for $8,000 so adding the unrecognized loss increases her basis to $13,000. By increasing basis in the amount of the unrecognized loss, Andrea will receive that back tax free when she ultimately sells the stock.

162
Q

Which of the following is correct?

Realized gains are always recognized and realized losses are never recognized.
Realized gains and realized losses on the sale of personal use assets are not recognized.
Realized gains and realized losses on the sale of personal use assets are recognized.
In an involuntary conversion, the date of realization, not the payment date, determines the date of recognition.

A

In an involuntary conversion, the date of realization, not the payment date, determines the date of recognition.
- Option “A” is incorrect because realized gains may, in some instances, be deferred or excluded such as the gain on sale of a personal residence.

Option “B” is incorrect because realized gains on some personal use assets are recognized unless excluded.

Option “C” is incorrect because realized losses on personal use assets are not recognized.

163
Q

On December 31 of last year, Uli purchased 100 shares of Runway, Inc. (a publicly held company) for $5,000. On March 1 of this year, Runway, Inc. declared that it was bankrupt, that it will wrap up operations, and that all of its assets will be used to satisfy secured creditor claims so there will be no residual equity left for the stockholders. Which of the following statements describes the tax treatment of this transaction?

Uli may deduct the $5,000 as an ordinary loss.
Uli may deduct the $5,000 investment as a short-term capital loss.
Uli may deduct the $5,000 investment as a long-term capital loss.
No loss deduction is permitted.

A

Uli may deduct the $5,000 investment as a short-term capital loss.
- Since the company became worthless during the year, a constructive sale of the stock occurs on December 31 of this year. Therefore, Uli has a short-term holding period because she is deemed to have held the stock for exactly one year (long-term holding period requires at least one year and one day). Therefore, Uli will be able to deduct the full $5,000 investment as a short-term capital loss in the year the company becomes worthless.

164
Q

Which of the following is a deductible loss?

Losses on the sale of personal use assets.
Losses on the subsequent sale of property gifted or sold to a related party when its fair market value is less than the original owner’s adjusted basis.
A wash sales transaction.
Capital losses in excess of $3,000.

A

Capital losses in excess of $3,000.
- Up to $3,000 of capital losses may be recognized against forms of income other than capital gains in any one tax year. However, if a taxpayer has capital losses in excess of $3,000, these losses are not disallowed, but are carried over indefinitely to future tax years.

165
Q

Penny had $44,000 of ordinary income. In addition, she had the following capital transactions during the last tax year: $1,800 net long-term capital loss, $1,800 net short-term capital loss. What is Penny’s capital loss carryover to the current tax year?

$0
$600 short-term.
$600 long-term.
$300 short-term and $300 long-term.

A

$600 long-term.
- Penny’s total capital loss for the previous tax year was $3,600. Assuming there were no capital gains that year, Penny would have taken the maximum capital loss of $3,000 leaving the remaining $600 to be deducted in the following year’s return. Short term losses are taken ahead of long term losses.

166
Q

The constructive receipt doctrine:

Applies to a secular trust used for deferred compensation.
Does NOT apply to cash basis taxpayers.
Is used to distinguish unearned income from earned income.
Means that a taxpayer cannot plan transactions to defer the recognition of income.

A

Applies to a secular trust used for deferred compensation.
- The cash method of accounting recognizes income when received. Receipt may be either actual or constructive. Constructive receipt is when the taxpayer has the right to the money although they are not in possession thereof. A secular trust constructively belongs to the beneficiary therefore constructive receipt applies.

167
Q

Arrange the following statutes of limitation from shortest to longest:

I. Collection of deficiency by the IRS.
II. Fraud.
III. General Statue of Limitations under Section 6501.
IV. Substantial Understatement of Income greater than 25%.

a) I, II, III, IV.
b) II, III, IV, I.
c) III, IV, I, II.
d) I, IV, III, II.

A

c) III, IV, I, II.
- The statute of limitations for the collection of a deficiency by the IRS is 10 years. There is no statute of limitations for fraud. The general statute of limitations under Section 6501 is 3 years. The statute of limitations for a substantial understatement of income greater than 25% is 6 years.

168
Q

Which of the following statements is/are NOT representative of the Federal income tax on individuals?

Federal Income tax rates are progressive.
The tax base for the application of income tax rates is taxable income.
Persons who have earned income other than wages are subject to withholdings.
All of the above are correct.

A

Persons who have earned income other than wages are subject to withholdings.
- All persons with taxable income are subject to the “pay-as-you-go” payment procedure but not withholdings. They must make estimated payments or face interest and penalties.

169
Q

Cindy owned her home six months prior to moving into it. She had lived in her home for 18 months when her employer required that she move to another state to manage its sales office. Her realized gain from the sale of her home is $149,000. Does Cindy have to report income, and if so, how much?

Cindy does not have to report income because she has owned and used her home for 18 months and her gain is less than the pro rata exemption.
Cindy reports a gain of $149,000 because she did not meet the owned and used rule.
Cindy may exclude up to 75% of her actual gain under the allowable exclusion because the move was job related.
Cindy may exclude up to 75% of the allowable exclusion but must purchase a replacement home within two years at a cost equal to or exceeding the selling price.

A

Cindy does not have to report income because she has owned and used her home for 18 months and her gain is less than the pro rata exemption.
- Section 121 requires that to qualify for the exemption she must have 1) owned and 2) used as principal residence for two years out of the past 5 years. An exception to the rules exists where a taxpayer moved because of employment transfer. In this case, Cindy owned and lived in the home for 18 of the 24 months. Cindy is entitled to 75% of the allowable $250,000 gain exemption or $187,500.

170
Q

Britney owned an office building in Los Angeles that she rented out to several production companies. The building was destroyed by a fire and was a complete loss. Britney received a settlement from her insurance company and would like to reinvest in a new property. Britney wants to make sure that she is eligible for nontaxable exchange treatment. Which of the following is not correct regarding the requirements for nontaxable exchange treatment on Britney’s transaction?

Because Britney rented out the building instead of using the property directly, the replacement property must meet the functional use test.
Britney must invest the proceeds in a replacement property that has a similar use to the property that was destroyed in the fire.
Britney must reinvest the insurance proceeds within two years from the end of the year in which she received the insurance proceeds.
Since Britney received cash as a result of the involuntary conversion, nonrecognition treatment is not mandatory even if she meets all of the requirements.

A

Because Britney rented out the building instead of using the property directly, the replacement property must meet the functional use test.
- The replacement property must meet the taxpayer use test, not the functional use test, since Britney did not use the property directly. The taxpayer use test requires replacement property to be used by the taxpayer in an activity which is treated the same for tax purposes in order to qualify for nontaxable exchange treatment. All of the other statements regarding the nontaxable exchange treatment of Britney’s transaction are correct.

171
Q

Which of the following statements regarding depreciation is/are correct?

I. Real estate used for residential rental purposes is depreciated on a straight-line basis over 27 1/2 years.
II. Real estate used for commercial purposes is depreciated on a straight-line basis over 39 years.
III. Both the land and the value of the improvements on the land can be depreciated.
IV. A mid-year convention is used in the depreciation of real property.

I and II only.
I, II and III only.
I, II, and IV only.
I, II, III, and IV.

A

I and II only.
- Statement “III” is incorrect because only the value of the improvements on the land can be depreciated. The value of the land itself cannot be depreciated. Statement “IV” is incorrect because a mid-month, not a mid-year convention is used in the depreciation of real property.

172
Q

What classifications of property are subject to cost recovery?

I. Personalty.
II. Personal use property.
III. Natural resources.
IV. Intangible property.
V. Real estate including land.

II and IV only.
I, III and IV only.
V only.
All of the above.

A

I, III and IV only.
- Personal use property is not subject to cost recovery since it is not used for income generating business purposes. Natural resources are subject to depletion and intangibles are subject to amortization. Personalty assets are used in business and are subject to depreciation. Real estate, as in permanent structures on land, are subject to cost recovery, but the land is not.

173
Q

Diane purchased a hotel on November 15, five and 1/4 years ago for $5,000,000. Determine the cost recovery deduction for one month.

$10,683
$15,152
$21,368
$128,200

A

$10,683
- To depreciate real property, the mid-month convention is used. In addition, a hotel is not considered residential real property and is therefore depreciated using straight line depreciation over 39 years

1 ÷ 39 = .02564

$5,000,000 × 2.564% = $128,200 of annual depreciation expense

$128,200 ÷ 12 = $10,683 for one month’s depreciation in the current year

174
Q

For tax purposes, a deduction is allowed for the consumption of the cost of an intangible asset through:

Depletion.
Depreciation/Cost Recovery.
Goodwill.
None of the above.

A

None of the above
- Cost recovery of an intangible asset is allowed through amortization. Cost recovery and depreciation (one in the same) are applied to tangible assets. The costs of natural resources are recovered through depletion.

175
Q

Capital recoveries include:

The initial outlay for capital improvements.
Repair and maintenance expenditures.
Salvage value.
Amortization of bond premium.

A

Amortization of bond premium.
- Capital recovery is the expensing of certain acquisition costs. Bonds purchased at a premium are amortized over their life to expense the premium paid. The theory is that when they mature, their basis will be equal to their face value and not the face plus premium. Bond expenditures are, therefore, a recovery of capital.

176
Q

Sara and Bill have rental property that was rented this year to a family whose primary bread winner lost his job. As a result they had uncollected rent for 2 months before they began the eviction process and 1 additional month before the family was finally evicted. If the rent was $800 per month, how much of a deduction may Sara and Bill claim on their income tax return for the uncollected rent?
None, uncollected rent is not deductible.
$800 of uncollected rent for the month it took to evict the renters.
$1,600 of uncollected rent before action was taken to evict the renters.
$2,400 of uncollected rent for the entire delinquency.

A

None, uncollected rent is not deductible.
- They cannot deduct from ordinary income an amount that was never included in taxable income. Therefore, Sara and Bill will not receive any deduction for the uncollected rents.

177
Q

James and Marilyn Herbert are married and own a vacation home on the beach in Florida. Each summer they are able to rent the property for $1,000 per week. This year they rented the property to six different parties and the total rental period was 133 days. James and his family vacationed there in the fall for three weeks. Expenses for the entire year totaled:

Mortgage Interest: $13,500

Mortgage Principal: $4,200

Real Estate Taxes: $6,400

Utilities: $2,300

Trash: $300

Management Fees: $3,800

Depreciation: $10,000

What is the Herbert’s profit or loss, and its nature, associated with this property for the current year?

$21,500 loss, not deductible.
$8,000 loss, deductible.
$12,350 loss, not deductible.
$2,414.87 loss, not deductible.

A

$2,414.87 loss, not deductible.
- Interest and taxes are accrued daily.

$13,500 + $6,400 = $19,900 × (133÷365) = $7,251.23

Other costs are deducted according to use time (133 + 21 = 154)

$16,400 × (133 ÷ 154) = $14,163.64

Total Costs $21,414.87 - Total Revenue $19,000 (133 ÷ 7 = 19 × $1,000) = Loss $2,414.87 not deductible due to mixed use.

178
Q

Mackenzie has two apartment units that are occupied by tenants all year long. In December, the tenants in unit 2 paid him in advance for the next January’s rent. The regular rent is $1,000 per month for each of the units. How much rental income must Mackenzie include in taxable income this year?

$12,000
$24,000
$25,000
It depends on which accounting method he uses

A

$25,000
- According to Publication 17, “Advance rent is any amount you receive before the period that it covers. Include advance rent in your rental income in the year you receive it regardless of the period covered or the method of accounting you use.”

179
Q

Payton owns farm land in west Texas where he raises cattle. In March of this year Austin approaches Payton about renting 25% of Payton’s land for purposes of growing wheat. Payton and Austin agree that Austin will only pay 3 months of rent at an amount of $8,000 per month if Austin will build a barn on the land, which is the equivalent to 6 months of rent. How much will Payton recognize as rental income this year?

$8,000
$24,000
$72,000
$96,000

A

$72,000
- Payton’s rental income is the cash received of $24,000 ($8,000 × 3) plus the fair market value of any property received. Since they agreed that 6 months of rent would equal the fair market value of the barn, the additional value is $48,000 ($8,000 × 6).

180
Q

A shareholder in an S corporation:

I. Must be a US citizen.
II. Votes for the Board of Directors at the annual shareholders’ meeting.
III. Receives a K-1 annually in order to prepare a personal income tax return.
IV. Reports on a personal income tax return a pro-rata share of corporate profit or loss.
I, II and III only.
I and III only.
II and IV only.
II, III, and IV only.

A

II, III, and IV only.
- Ownership of S corporation stock is restricted to individuals who are US citizens OR residents, estates, certain trusts, and charitable organizations. A shareholder in an S corporation may vote to retain or revoke S corporate status, votes, receives an annual K-1 and reports their pro rata share of profit or loss on their personal income tax return.

181
Q

David is a wealthy attorney in the highest marginal income tax bracket. He is interested in purchasing a franchise in a fast-growing food chain with some of his associates. After reviewing the proposal, you have determined that apart from a substantial up-front investment, the business will NOT need to retain income and any income generated in subsequent years will be paid out to the investors.

David wants to be assured the business would NOT be disrupted if one of his associates lost interest or encountered personal financial reversals. What form of business structure would make the most sense?

A Limited Partnership.
A General Partnership.
A C Corporation.
A S Corporation.

A

A S Corporation.
- Options “A” and “B” are incorrect in that they would dissolve in the event of a 50% turnover in ownership in a 12-month period. Option “C” is incorrect because a C corporation generally retains income. Option “D” is correct as an S corporation holds no retained earnings since all profits or losses are passed through to its owners.

182
Q

Brady is starting a new business where he will be the sole owner. He would like to have limited liability, but he would prefer flow-through taxation because he expects to have losses in the first few years. He is not concerned about incurring self-employment taxes. Which of the following entities would best suit Brady’s needs?

A General or Limited Partnership.
A Single-member LLC.
A S corporation.
A Proprietorship.

A

A Single-member LLC
- Both the proprietorship and the single-member LLC would provide the flow-through taxation that Brady is looking for, but only the LLC would also provide limited liability.

183
Q

Which of the following are characteristics of a C corporation?

I. The number of shareholders is unlimited.
II. The bankruptcy of a major shareholder has NO effect on the business form.
III. Shareholder liability is limited.
IV. The capital structure is dependent upon the resources of the shareholders.

I and II only.
III and IV only.
I, II, and III only.
I, II, III and IV.

A

I, II, and III only.
- Options “II” and “III” are accurate. The number of shareholders is limited in an S corporation, but not in a C corporation. Capital may be raised by the sale of stock and by borrowing in capital markets.

184
Q

Larry, Mike and Clarence wish to start a manufacturing business. The business is expected to generate a large income which it will reinvest for many years. Larry has substantial assets which he plans to contribute to the business. Larry is also concerned about showing too much business income on his personal return. Which of the following structures would serve as an appropriate form of organization to satisfy Larry, Mike and Clarence’s objective?

I. A limited partnership with Larry as the limited partner.
II. A business trust with all three as equal interests.
III. An S corporation with all three as equal shareholders.
IV. A C corporation with all three as shareholders.

I, II and III only.
I and IV only.
II and IV only.
IV only.

A

IV only.
- Options “I,” “II” and “III” are all pass through entities which will require the owner to report on their personal income tax return their pro rata share of income. Therefore, Options “I,” “II” and “III” are all incorrect. The C corporation will not expose the shareholders to their share of undistributed corporate income.

185
Q

Sabina Herz, a school teacher who is in a low marginal income tax bracket and is interested in investing in an educational business with some of her colleagues. Sabina says that she wants to keep the money in the business and doesn’t want to pay taxes on income she never sees. Furthermore, Sabina wants to be assured the business would NOT be disrupted if one of her partners lost interest or encountered personal financial problems. What legal form of business makes the most sense, given Sabina’s desires?

A Limited Partnership.
A General Partnership.
A C Corporation.
A Professional Corporation.

A

A C Corporation.
- Options “A” and “B” are incorrect in that they would dissolve in the event of a 50% turnover in ownership in a 12-month period. Option “C” is correct because a C corporation generally retains income that is not passed to its owners through dividends. Option “D” is incorrect because it is structured as a C corporation for professional services. Not an educational business.

186
Q

To which of the following entities does the term “pass-through” entity apply?

I. LLC.
II. C corporation.
III. Not for Profit Corporation.
IV. S corporation.
V. Partnership.

a) I, III and IV only.
b) I, IV and V only.
c) I, III, IV and V only.
d) II and III only.

A

b) I, IV and V only.
- The S corporation, LLC and Partnership are considered “pass through” entities. “Pass through” means that the entity is not taxed separately from its owners, but passes its profits and losses through to the owners in their pro rata share of ownership.

187
Q

Which of the following statements concerning the Alternative Minimum Tax (AMT) system is correct?

Deferral items for AMT purposes result in a permanent increase in tax.
Exclusion items for AMT purposes may be reversed in future tax years.
There is an unlimited carryforward for AMT credit generated in a year the taxpayer becomes an AMT taxpayer.
The AMT credit can be carried back for up to two years, provided that use of the credit does not force the taxpayer to become an AMT taxpayer in those years.

A

There is an unlimited carryforward for AMT credit generated in a year the taxpayer becomes an AMT taxpayer.
- The AMT credit that is generated from deferral items in the AMT calculation may be carried forward indefinitely. The AMT credit may not be carried back. Deferral items may be reversed in future years through the use of the AMT credit, while exclusion items result in a permanent increase in tax.

188
Q

Which of the following are preference items or adjustments for purposes of the individual alternative minimum tax?

Qualified private-activity municipal bond interest.
The excess of percentage depletion over the property’s adjusted basis.
Investment interest in excess of net investment income.
Qualified housing interest.
a) I only.
b) II and IV only.
c) I and II only.
d) II and III only.

A

c) I and II only.
- By definition, investment interest expense in excess of net investment income and qualified housing interest are not preference items or adjustments for purposes of the alternative minimum tax.

189
Q

Tommy Vasquez has taxable income of $200,000. He is concerned about being subject to the alternative minimum tax (AMT). The following income and deductions were included in computing his taxable income. Select one item which may be added to (or subtracted from) regular taxable income in calculating the AMT.

A long-term capital gain of $100,000.
A state income tax deduction of $5,000.
Dividend income of $100,000.
Personal Residence interest deduction of $25,000

A

A state income tax deduction of $5,000.
- Options “A”, “C” and “D” are incorrect as they are neither added nor subtracted from the regular income to calculate AMT. State income taxes deducted as an itemized deduction will be added back into regular income to calculate Alternative Minimum Taxable Income.

190
Q

Which of the following is not an add-back item for purposes of calculating the Alternative Minimum Tax (AMT)?

Depreciation of property placed in service after 1986.
The standard deduction, if taken in lieu of itemized deductions.
Passive activity losses.
Installment sales undertaken before March 1, 1986.

A

Installment sales undertaken before March 1, 1986.

- Installment sales executed after March 1, 1986 but not before, are add-back items.

191
Q

All of the following statements concerning the AMT as it applies to individual taxpayers are correct EXCEPT:

Some itemized deductions taken for regular tax purposes must be added back to regular income to determine income under AMT.
Taxpayers are permitted to take the standard deduction for both regular and AMT tax purposes.
All adjustments made to itemized deductions when calculating AMT result in a permanent increase in tax.
Charitable deductions are claimed in the same manner for regular tax and for AMT tax purposes.

A

Taxpayers are permitted to take the standard deduction for both regular and AMT tax purposes.
- Taxpayers who do not itemize deductions take the standard deduction for regular tax purposes, but this is added back to alternative minimum taxable income for AMT purposes. All of the other statements are correct.

192
Q

Giselle became an AMT taxpayer last year. She had to add several items to her regular taxable income in arriving at alternative minimum taxable income. Which of the following items will result in an AMT credit that can be used to offset future regular tax liability?

$5,000 in property taxes paid on her principal residence if paid in advance.
A $75,000 difference between the fair market value of stock and the strike price in the incentive stock option used to purchase the stock.
$4,000 in interest on private activity municipal bonds.
$4,000 in additional medical expenses.

A

A $75,000 difference between the fair market value of stock and the strike price in the incentive stock option used to purchase the stock.
- The inclusion of the difference between the fair market value and exercise price of the stock options will result in a credit that Giselle can use against future regular income tax liability. The other items are adjustments made to her itemized deductions, which result in permanent differences in tax liability as a result of the imposition of the AMT.

193
Q

During the current year, JoAnne’s business made net income before any Section 179 deduction of $15,000. She added an allowable Section 179 asset to her business valued at $150,000 as of the first of the current year. What can she deduct under Section 179 in the current year?

$25,000
$15,000
$135,000
$150,000

A

$15,000
- Section 179 deduction cannot exceed net business income in a given year. The rest of the Section 179 deduction can be carried over, but a 179 deduction cannot be used to create a business loss. Therefore, they would only be allowed to deduct $15,000 for the current year and carry forward $135,000.

194
Q

When do the recapture rules for Section 179 apply:

I. When the asset is sold before it would have been fully depreciated.
II. When the business use drops below 50%.
III. When the Section 179 deduction taken in one year exceeds the allowable maximum.
IV. When there is sufficient income in one tax year to support the deduction taken.
V. When the deduction causes the tax liability to become negative.

III and IV only.
I and II only.
V only.
All of the above.

A

I and II only.
- Section 179 recapture rules apply when the business use of an asset drops below 50% for a given year or when the asset is disposed of before it would have been fully depreciated.

195
Q

What is the primary advantage of using the Section 179 Deduction over other cost recovery methods?

By deducting more currently, total tax liability is reduced and the present value of cash flows is increased.
The $1,020,000 Section 179 limit allows a businesses to deduct more up front.
Section 179 reduces the depreciation on most assets to only three years.
Depreciation applies only to business assets, whereas Section 179 applies to business and personal assets.

A

By deducting more currently, total tax liability is reduced and the present value of cash flows is increased.
- Section 179 is an upfront business deduction, now at $1,020,000 (2019) that can be used by businesses to reduce tax liabilities. It’s possible to reduce Section 179 deduction to zero, depending how much is placed into service. If too much is placed into service, Section 179 would not have any advantages over other methods of depreciation.

196
Q

Kal has taxable income this year of $6 million. He purchased $2,584,000 worth of depreciable property this year and is trying to calculate his §179 deduction. What is the correct amount?

$466,000
$530,000
$986,000
$1,020,000

A

$986,000
- Since he placed into service more assets than allowed under the limitation you must calculate the phase-out.

2,584,000 placed into service less 2,550,000 placed into service limit = 34,000

$1,020,000 - $34,000 = $986,000.

Since this is below the net income of the business there are no further limitations.

197
Q

Kevin’s 12 year old daughter, Angel, has a brokerage account that generates $13,000 of interest income and $2,000 of qualified dividends for the current year. Angel also has earned income of $13,000 from modelling that she is saving for college. How much will be taxed at Angel’s tax rate?

$1,100
$3,000
$12,800
$15,800

A

$3,000
- The calculation is as follows:

Start with figuring the standard deduction, earned income +350 not to exceed 12,200 in 2019.

Unearned Income

13,000 + 2,000 = 15,000

15,000 - 1,100 (standard deduction) = 13,900

13,900 - 1,100 (at the child’s rate) = 12,800

The remaining 12,800 will be taxed at the trust and estate rate.

Earned Income

13,000 - 11,100 (remaining standard deduction: 12,200 - 1,100) = 1,900 at the child rate.

Summary:

Standard deduction is 1,100 + 11,100 = 12,200

At the trust rate 12,800

At the child’s rate is 1,100 + 1,900 = 3,000

198
Q

Two years ago, Green Corporation, a cash basis taxpayer, sold services to Albert for $25,000. During the prior year, Green collected $10,000 from Albert. In the current year, Green collected $5,000 from Albert in final settlement of the debt. The proper treatment for the bad debt deduction is:

$0 for the prior year and $0 for the current year.
$0 for the prior year and $10,000 for the current year.
$15,000 for the prior year and $0 for the current year.
$15,000 for the prior year and $5,000 income for the current year.

A

$0 for the prior year and $0 for the current year.
- A cash basis taxpayer does not recognize income not received. Since the bad debt was never recognized as income, it cannot be recognized as a loss or a bad debt expense.